National Institute of Fashion & Technology (NIFT) Post Graduate 2020 Question Paper With Answer Key

NIFT (Post Graduate)

National Institute of Fashion and Technology

Solver Paper 2020

1. Three Science classes A, B and C take a Life Science test. The average score of class A is 83. The average score of class B is 76. The average score of class C is 85. The average score of classes A and B is 79 and average score of classes B and C is 81. Then, the average score of classes A, B and C is

(a)  80

(b)  80.5

(c)  81.5

(d)  81

Answer: (c)

2. In a library the ratio of story books and other books is 7 : 2 and there are 1512 story books. Due to collection of some more story books the said ratio becomes 15 : 4. The number of story books collected is

(a)  100

(b)  97

(c)  108

(d)  205

Answer: (c)

3. A and B have their monthly incomes in the ratio 8 : 5, while their monthly expenditures are in the ratio 5 : 3. If they have saved Rs. 12000 and Rs. 10000 monthly respectively, then the difference in their monthly incomes is

(a)  Rs. 52000

(b)  Rs. 46000

(c)  Rs. 44000

(d)  Rs. 42000

Answer: (d)

4. A farmer travelled a distance of 61 km in 9 h. He travelled partly on foot at the rate 4 km/h and partly on bicycle at the rate 9 km/h. The distance travelled on foot is

(a)  14 km

(b)  15 km

(c)  16 km

(d)  17 km

Answer: (c)

5. A sum of Rs. 7930 is divided into 3 parts and given on loan at 5% simple interest to A, B and C for 2 yr, 3 yr and 4 yr, respectively. If the amounts of all three are equal after their respective periods of loan, then the A received a loan of

(a)  Rs. 2760

(b)  Rs. 3050

(c)  Rs. 2800

(d)  Rs. 2750

Answer: (a)

6. A and B can do a piece of work in 30 and 36 days, respectively. They began the work together but A leaves after some days and B finished the remaining work in 25 days. After how many days did A leave?

(a)  6 days

(b)  11 days

(c)  10 days

(d)  5 days

Answer: (d)

7. From 1980-90, the population of a country was increased by 20%. From 1990-2000, the population of the country was increased by 20%. From 2000-10, the population of the country was increased by 20%. Then the overall increased population (in percentage) of the country from 1980-2010 was

(a)  60%

(b)  62.8%

(c)  72.2%

(d)  72.8%

Answer: (d)

8. If the product of two positive numbers be 1575 and their ratio is 7 : 9, then the greater number is

(a)  35

(b)  63

(c)  45

(d)  135

Answer: (c)

9. If A : B = 2 : 3 and B : C = 3 : 7, then

A + B : B + C : C + A is equal to

(a)  4 : 10 : 9

(b)  5 : 8 : 9

(c)  5 : 10 : 9

(d)  4 : 8 : 9

Answer: (c)

10. The HCF and LCM of two numbers are 21 and 84, respectively. If the ratio of the two numbers is 1 : 4, then the larger of the two numbers is

(a)  84

(b)  108

(c)  48

(d)  12

Answer: (a)

11. A sell a cycle to B at a profit of 20% and B sells it to C at a loss of 25%. If C bought the cycle for Rs. P, then the cost price of it for A was

(a)  Rs. 9/10 P

(b)  Rs. 10/9 P

(c)  Rs. 1/20P

(d)  Rs. 9/20 P

Answer: (b)

12. A number when divided by 361 gives a remainder 47. If the same number is divided by 19, then the remainder obtained is

(a)  8

(b)  1

(c)  3

(d)  9

Answer: (d)

13. If 64 buckets of water are removed from a cubical shaped water tank completely filled with water, 1/3 of the tank remains filled with water. The length of each side of the tank is 1.2 m. Assuming that all buckets are of the same measure, then the volume (in litre) of water contained by each bucket is

(a)  12

(b)  16

(c)  18

(d)  15

Answer: (c)

14. In a farm there are cows and hens. If heads are counted there are 180, if legs are counted there are 420. The number of cows in the farm is

(a)  150

(b)  30

(c)  130

(d)  50

Answer: (b)

15. If the arithmetic means of 3a and 4b is greater than 50, and a is twice of b, then the smallest possible integer value of a is

(a)  18

(b)  19

(c)  20

(d)  21

Answer: (d)

16. If the average of eight consecutive even number be 93, then the greatest number among them is

(a)  86

(b)  98

(c)  100

(d)  102

Answer: (c)

17. x can copy 80 pages in 20 h, x and y together can copy 135 pages in 27 h. Then y can copy 20 pages in

(a)  3 h

(b)  12 h

(c)  20 h

(d)  24 h

Answer: (c)

18. The average of the largest and smallest 3 digit number formed by 0, 2 and 4 would be

(a)  213

(b)  303

(c)  312

(d)  222

Answer: (c)

19. In what ratio two types of tea costing Rs. 180/kg and Rs. 280/kg should be mixed in the ratio so that the mixture obtained sold at Rs. 320/kg to earn a profit of 20% is

(a)  1 : 13

(b)  2 : 13

(c)  3 : 13      

(d)  4 : 13

Answer: (b)

20. If a person marked a product 25% above the cost price but allows 10% discount, then the percentage of profit is

(a)  15%

(b)  12.5%

(c)  35%

(d)  17.5%

Answer: (b)

21. x and Mr. y each bought the same motorcycle using a 10% off coupon. Mr. x’s cashier took 10% off the price and then added 8.5% sales tax, whereas Mr. y’s cashier first added the sales tax and then took 10% off the total price. The amount Mr. x paid is

(a)  same as the amount Mr. y paid

(b)  greater than Rs. 850 as the amount Mr. y paid

(c)  less than Rs. 550 as the amount Mr. y paid

(d)  greater than Rs. 85 as the amount Mr. y paid

Answer: (a)

22. Two workers A and B are engaged to do a piece of work A working alone would take 8 h more to complete the work than when work together. If B worked alone, would take  more than when work together. The time required to finish the work together is

(a)  4 h

(b)  6 h

(c)  5 h

(d)  8 h

Answer: (b)

23. A and B invest in the ratio 3 : 5. After 6 months, C joins the business investing an amount equal to amount of B. At the end of the year what will be the ratio of their profits?

(a)  3 : 5 : 2

(b)  3 : 5 : 5

(c)  6 : 10 : 5

(d)  8 : 10 : 5

Answer: (c)

24. If a man walks at the rate of 5 km/h, he misses a train by 7 min. However, if he walks at the rate of 6 km/h, he reaches the station 5 min before the arrival of the train. The distance covered by him to reach the station is

(a)  6 km

(b)  4 km

(c)  7 km

(d)  6.25 km

Answer: (a)

25. A dealer fixed the price of an article 40% above the cost of production. While selling it he allows a discount of 20% and makes a profit of Rs. 48. The cost of production (in Rs.) of the article is

(a)  400

(b)  360

(c)  320

(d)  420

Answer: (a)

26. Articles are marked at a price which gives a profit of 25%. After allowing a certain discount the profit reduces to  The discount per cent is

(a)

(b)  12%

(c)  10%

(d)  11.1%

Answer: (c)

27. The average age of 30 students of a class is 14 yr 4 months. After admission of 5 new students in the class the average becomes 13 yr 9 months. The youngest one of the five new students is 9 yr 11 months old. The average age of the remaining 4 new students is

(a)  13 yr 6 months

(b)  12 yr 4 months

(c)  10 yr 4 months

(d)  11 yr 4 months

Answer: (c)

28. A sum of money is paid back in two annual installments of Rs. 17640 each, allowing 5% compound interest compounded annually. The sum borrowed was

(a)  Rs. 32400

(b)  Rs. 32000

(c)  Rs. 32200

(d)  Rs. 32800

Answer: (d)

29. There is a number consisting of two digits, the digit in the unit’s place is twice that in the ten’s place and if 2 is subtracted from the sum of the digits, the difference is equal to 1/6th of the number. The number is

(a)  26

(b)  23

(c)  25

(d)  24

Answer: (d)

30. Average of n numbers is a. The first number is increased by 2, second one is increased by 4, the third one is increased by 8 and so on. The average of the new numbers is

Answer: (b)

Directions (Q. Nos. 31-34) In each of the following questions, out of the given alternatives, choose the one which best expresses the meaning of the given word.

31. ENDORSEMENT

(a)  Reprimand

(b)  Censure

(c)  Commendation

(d)  Reproach

Answer: (c)

32. COMMEMORATE

(a)  Boast

(b)  Remember

(c)  Manipulate

(d)  Harmonise

Answer: (b)

33. WHIM

(a)  Fancy

(b)  Clumsy

(c)  Desire     

(d)  Strange behaviour

Answer: (c)

34. CANDID

(a)  Rude

(b)  Impertinent

(c)  Blunt

(d)  Frank

Answer: (d)

Directions (Q. Nos. 35-38) In each of the following questions, a word has been spelt in four different ways out of which only one is correct. Find the correctly spelt word.

35.

(a)  Mountaineous

(b)  Mountaneous

(c)  Mountainous

(d)  Mountanous

Answer: (c)

36.

(a)  Hindrence

(b)  Hindrane

(c)  Hinderence

(d)  Hinderance

Answer: (b)

37.

(a)  Sacrosanct

(b)  Sacrosenct

(c)  Secrosanct

(d)  Sacrosantt

Answer: (a)

38.

(a)  Palid

(b)  Pallid

(c)  Palled

(d)  Pellid

Answer: (b)

Directions (Q. Nos. 39-42) In each of the following below four sentences are given which are denoted by (A), (B), (C), (D). By using all the four sentences you have to frame a meaningful paragraph. The correct order of the sentences is your answer. Choose from the four alternatives the one having the correct order of sentences and mark it as your answer.

39. (A) Now under liberated economy they are learning to compete domestically and globally.

(B) In India corporations until recently achieved success by avoiding competition, using protected and regulated domestic markets.

(C) The trend is irreversible.

(D) Business leaders are preparing themselves to meet competitive challenges, and to avoid being swept away.

(a)  BADC

(b)  BDCA

(c)  BDAC

(d)  CDBA

Answer: (a)

40. (A) Recovery was given inadequate attention and consequently some bank branches regularly incurred heavy losses and their parent bodies had to bale them out.

(B) As a result, banks indulged in extensive lending to borrowers who had little or no potential to make repayments.

(C) To fulfill the social objectives laid down by the master of nationalization, banks were asked to lend to identified priority sectors.

(D) 1992-93 results showed that the loss making branches of public sector banks increased from 10,000 to 13,000 and the quantum of losses showed at Rs. 3,369 crores.

(a)  BACD

(b)  DABC

(c)  CBAD

(d)  BCAD

Answer: (c)

41. (A) Finally the bureaucratic organization took over from the pioneering enterprise.

(B) The nineteenth century was the age of entrepreneur, the self-made man.

(C) Thoughtful business administration took over from action-centred business

(D) In the twentieth century the rational executive took command.

(a)  DBAC

(b)  CABD

(c)  BDCA

(d)  BCDA

Answer: (c)

42. (A) He was so busy with them that he did not get time to eat.

(B) Thousands of people came to him and asked different types of questions.

(C) No one cared to see that he had his food or rest that night.

(D) Swami Vivekanand once stayed in a small village.

(a)  BCDA

(b)  CBAD

(c)  DBAC

(d)  DBCA

Answer: (c)

Directions (Q. Nos. 43-47) Each item in this section consists of sentences with an underlined word followed by four words or group of words. Select the option that is opposite in meaning to the underlined word.

43. He speaks eloquently and can pull crowds.

(a)  confusingly

(b)  expressively

(c)  powerfully

(d)  fluently

Answer: (a)

44. Everyone has to fight the inertia in the system.

(a)  sluggishness

(b)  indolence

(c)  activity

(d)  torpor

Answer: (c)

45. There is a need to promote philanthropy in education.

(a)  charity

(b)  benevolence

(c)  nastiness

(d)  likeliness

Answer: (c)

46. What we lack in the current times is

(a)  empathy

(b)  carefulness

(c)  indifference

(d)  hardship

Answer: (c)

47. Tempestuous behavior would not yield much in any place.

(a)  relaxed

(b)  passionate

(c)  intense

(d)  windy

Answer: (a)

Directions (Q. Nos. 48-52) Each of the following sentences in this section has a blank space and four words or group of words given after the sentence. Select the word or group you consider most appropriate for the blank space.

48. The Election Commission on Saturday …….. that the Assembly elections in five states will be held from 12th November to 7th

(a)  pronounced

(b)  announced

(c)  promulgated

(d)  issued

Answer: (b)

49. The victims were fruit vendors and they were going in an auto when they ……….. an accident on the way.

(a)  met with

(b)  ran into

(c)  experienced

(d)  heard

Answer: (a)

50. Scores of villagers are …….. a sit-in protest against the construction of a new underpass.

(a)  performing

(b)  sitting

(c)  staging

(d)  standing

Answer: (c)

51. It is common for patients to stop ………… medicine as soon as they start feeling better.

(a)  eating

(b)  gulping

(c)  swallowing

(d)  taking

Answer: (d)

52. A four-year-old girl got a new lease of life after doctors at a hospital ………… a cancerous tumour from one of her kidneys.

(a)  rejuvenated

(b)  removed

(c)  displaced

(d)  replaced

Answer: (b)

Directions (Q. Nos. 53-55) Choose the correct plural form of the given words.

53. Loaf

(a)  Loaves

(b)  Loafs

(c)  Loavs

(d)  Lofes

Answer: (a)

54. Spy

(a)  Spis

(b)  Spys

(c)  Spies

(d)  Spiess

Answer: (c)

55. Vertex

(a)  Vertics

(b)  Vertexs

(c)  Verticess

(d)  Vertices

Answer: (d)

Directions (Q. Nos. 56-75) Read the following passages and answer the questions that follow.

PASSAGE 1

Indian nationalism emanated from ‘traditional patriotism’, a socially active sentiment of attachment to land, language and cult, that developed in the subcontinent long before the process of westernization had begun. In India of the 18th and early 19th centuries, such sentiments were emerging on a regional basis as homeland was being defined by various terms like desh, vatan or nadu. But although regionally centred at Bengal, Maharashtra, Awadh or Mysore, their isolation broke down through various means of communication, the political legitimacy of  the Mughal empire was recognized throughout Hindustan, which was thought to be the abode of both the Hindus and the Muslims. Cultural barriers melted down through commercialization and regular pilgrimages. It burst forth through numerous acts of resistance; participated by both princes and the commoners, culminating in the Revolt of 1857. After the revolt, a modern sector of politics gradually evolved in India, through rapid spread of education, development of communication systems, such as the railways and telegraph and the emergence of a new public space created by the colonial institutions.

56. Which of the following was the result of the East India Company’s rule in India?

(a)  Good government

(b)  Consolidation of patriotic sentiments

(c)  New ethical tradition

(d)  Evangelicalism

Answer: (b)

57. Which of the following contributed most to the dissolution of cultural barriers?

(a)  Pilgrimage

(b)  Creation of Mughal Empire

(c)  Commercialization

(d)  Communication networks

Answer: (c)

58. In pre-colonial India, what did the nationalism mean?

(a)  Cultural construction of homeland

(b)  Linguistic affinity

(c)  Patriotism

(d)  Social activism

Answer: (d)

PASSAGE 2

Anxiety and conscience are a pair of powerful dynamos. Of course, I can only speak for myself between them, they have ensured that one shall work at anything worthwhile. They are blind forces which drive but do not direct. Fortunately, I have also been moved by a third motive-the wish to see and understand. Curiosity is another motive for action. It is also one of the distinctive characteristics of human nature (keep) contrasted with the natures of non-human animals.

All human beings have curiosity in some degree and we also have it about things which are of no practical use. Curiosity may be focused on anything in the universe, but the spiritual reality of the phenomenon should be the ultimate objective of all curiosity for it to be fruitful. Thanks to my mother, my approach to this ultimate objective is through the story of human affairs.

59. What is the main objective of this passage?

(a)  Distinguish between human beings and animals

(b)  Project curiosity as a potent motivating factor

(c)  Project anxiety and conscience as inadequate motivators

(d)  Motivation and spiritual reality

Answer: (b)

60. A characteristic, peculiar to human beings that is referred to, is

(a)  superior intelligence

(b)  spirit of enquiry

(c)  capacity to rationalize and analyze

(d)  ability to shift the practical from the impractical

Answer: (b)

61. Which of the following statements is true according to the passage?

(a)  Animals are as curious as human beings

(b)  Curiosity is the only motive for action

(c)  People motivated by curiosity do not need other motives to guide them

(d)  People motivated by anxiety and conscience alone can be misdirected

Answer: (d)

62. According to the author

(a)  those who have little curiosity are curious about unimportant things

(b)  apart from humans no other living being have the gift of curiosity

(c)  the highest form of the curiosity can be satisfied by study of human affairs alone

(d)  spiritual reality is the ultimate goal of humans through action

Answer: (d)

63. The author subscribes to the view that

(a)  one’s curiosity should focus beyond the façade on the latent meaning of things

(b)  curiosity is an inherited family characteristic

(c)  a study of human affairs is the most effective method of satisfying one’s curiosity

(d)  in order to motivate, curiosity must be coupled with anxiety and conscience

Answer: (a)

PASSAGE 3

One simple physical concept lies behind the formation of the stars : gravitational instability. The concept is not new. Newton first perceived it late in the 17th century.

Imagine a uniform, static cloud of gas in space. Imagine then that the gas is somehow disturbed so that one small spherical region becomes a little denser than the gas around it so that the small region’s gravitational field becomes slightly stronger. It now attracts more matter to it and its gravity increases further, causing it to begin to contract. As it contracts, its density increases, which increase its gravity even more, so that it picks up even more matter and contracts even further. The process continues until the small regions of gas finally forms a gravitationally bound object.

64. The primary purpose of the passage is to

(a)  describe a static condition

(b)  support a theory considered outmoded

(c)  depict the successive stages of a phenomenon

(d)  demonstrate the evolution of the meaning of a term

Answer: (c)

65. It can be inferred from this passage that the author views the information contained within it as

(a)  lacking in elaboration

(b)  original but obscure

(c)  speculative and unprofitable

(d)  uncomplicated and traditional

Answer: (d)

66. With which of the following words can you replace the word ‘uniform’ as given in this passage?

(a)  Uniting

(b)  Varying

(c)  Gaseous

(d)  Unvarying

Answer: (d)

67. What does the word ‘It in bold type stand for in the passage?

(a)  Gravitational instability

(b)  Cloud of gas

(c)  Small spherical denser region

(d)  Matter

Answer: (c)

68. The author provides information that answers which of the following questions? Choose the correct option.

(i) What causes the disturbances that changes the cloud from its original static condition?

(ii) How does this small region’s increasing density affect its gravitational field?

(iii) What is the end result of the gradually increasing concentration of the small region of gas?

(a)  Only (i)

(b)  Only (ii)

(c)  (ii) and (iii)

(d)  All of these

Answer: (c)

PASSAGE 4

As heart disease continues to be the number one killer in the United States, researchers have become increasingly interested in identifying the potential risk factors that trigger heart attacks. High fat-diets and life in the fast lane have long been known to contribute to the high incidence of heart failure. But according to new studies, the list of risk factors may be significantly longer and quite surprising. Heart failure, e.g., appears to have seasonal and temporal patterns. A higher percentage of heart attacks occurs in cold weather and more people experience heart failure on Monday than on any other day of the week.

In addition. People are more susceptible to heart attacks in the first few hours after waking. Cardiologists first observed this morning phenomenon in the mid-1980 and have since discovered a number of possible causes. An early morning rise in blood pressure, heart rate and concentration of heart stimulating hormones, plus a reduction of blood flow to the heart, may all contribute to the higher incidence of heart attacks between the hours of 8:00 am and 10:00 am. In other studies, both birthday and bachelorhood have been implicated as risk factors.

Statistics reveal that heart attack rate increases significantly for both females and males in the few days immediately preceding and following their birthdays. And unmarried men are more at risk for heart attacks than their married counterparts. Though, stress is thought to be linked in some way to all of the aforementioned risk factors, intense research continues in the hope of future comprehending why and how heart failure is triggered.

69. What does the passage mainly discuss?

(a)  Cardiology

(b)  Diet and stress as factors in heart attacks

(c)  Seasonal and temporal patterns of heart attacks

(d)  Risk factors in heart attacks

Answer: (c)

70. The word ‘potential’ could best be replaced by which of the following?

(a)  Harmful

(b)  Possible

(c)  Unknown

(d)  Primary

Answer: (b)

71. According to the passage, which of the following is not a possible cause of morning heart attacks?

(a)  Decreased blood flow to the heart

(b)  Increased blood pressure

(c)  Lower heart rate

(d)  Increase in hormones

Answer: (c)

72. Which of the following is cited as possible risk factor?

(a)  Getting married

(b)  Rainy weather

(c)  Eating fatty foods

(d)  Driving fast

Answer: (c)

73. Which of the following does the passage infer?

(a)  We now fully understand how the risk factors trigger heart attacks.

(b)  We do not fully understand how the risk factors trigger heart attacks.

(c)  We have not identified risk factors associated with heart attacks.

(d)  We have recently begun to study how risk factors work.

Answer: (b)

74. What could be the suitable title of the passage?

(a)  Heart disease in the USA

(b)  Causes of Heart Attack

(c)  Stress

(d)  Risk of Heart attack in Humans

Answer: (b)

75. Which word can be the most opposite word of ‘unlikely’?

(a)  Susceptible

(b)  Stimulating

(c)  Implicated

(d)  Aforementioned

Answer: (a)

Directions (Q. Nos. 76-80) Study the following information carefully and answer the given questions.

Eight friends, P, Q, R, S, T, V, W and Y are sitting around a square table in such a way that four of them sit at four corners of the square while four sit in the middle of each of the four sides. The ones who sit at the four corners face the centre while those who sit in the middle of the sides face outside.

P, who faces are centre, sits third to the right of V. T, who faces the centre, is not an immediate neighbor of V. Only one person sits between V and W. S sits second to right of Q. Q faces the centre. R is not an immediate neighbor of P.

76. Who sits second to the left of Q?

(a)  V

(b)  P

(c)  T

(d)  Y

Answer: (b)

77. What is the position of T with respect to V?

(a)  Fourth to the left

(b)  Second to the left

(c)  Third to the left

(d)  Third to the right

Answer: (c)

78. Three of the following four are alike in a certain way and so form a group. Which is the one that does not belong to that group?

(a)  R

(b)  W

(c)  V

(d)  S

Answer: (d)

79. Which of the following will come in place of the question mark(?) based upon the given seating arrangement?

WP TR QW RS ?

(a)  YT

(b)  VY

(c)  VQ

(d)  PY

Answer: (a)

80. Which of the following is true regarding R?

(a)  R is an immediate neighbor of V

(b)  R faces the centre

(c)  R sits exactly between T and S

(d)  Q sits third to left of R

Answer: (c)

81. In the given figure, which letter represents those Actors who are also Dancers, Singers as well as Violinists?

(a)  S

(b)  Q

(c)  P

(d)  U

Answer: (b)

Directions (Q. Nos. 82 and 83) In the following questions, which one of the following diagram represents the correct relationship among :

82. Lion, Fox and Carnivorous

Answer: (c)

83. Manager, Labour Union and Worker

Answer: (a)

84. In a certain code language ‘INSTITUTION’ is written as ‘NOITUTITSNI’. How will ‘PERFECTION’ is written in that language?

(a)  NOICTEFREP

(b)  NOITCEFERP

(c)  NOITCEFRPE

(d)  NOITCEFREP

Answer: (d)

85. In a certain code language ‘RELATED’ is written as ‘EFUBKDQ’, then how will ‘RETAINS’ be written in that language?

(a)  SDQBTOJ

(b)  JOTBQDS

(c)  JOTBSDQ

(d)  TOJBSDQ

Answer: (d)

Directions (Q. Nos. 86-89) In these questions, select the related letters from the given alternatives.

86. KNQT : LORU : : ADGJ : ?

(a)  BEHK

(b)  FHLO

(c)  DGEF

(d)  MPVW

Answer: (a)

87. DRIVEN : NEVIRD : : BEGUM : ?

(a)  MEUBG

(b)  BGMUE

(c)  EBGMU

(d)  MUGEB

Answer: (d)

88. ADIP : DGLS : : BEJQ : ?

(a)  EHMT

(b)  EJQU

(c)  CGLS

(d)  FINU

Answer: (a)

89. ACE : BDF : : CEG : ?

(a)  DEF

(b)  DFH

(c)  DEH

(d)  DFE

Answer: (b)

Direction (Q. No. 90) A series is given with one term missing. Choose the correct alternative from the given ones that will complete the series.

90. 

Answer: (c)

91. Fatima while introducing Mustafa to her husband said, “His brother’s father is the only son of my grandfather”. How is Fatima related to Mustafa?

(a)  Aunt

(b)  Sister

(c)  Niece

(d)  Mother

Answer: (b)

92. A, Q, Y and Z are different persons. Z is the father of Q. A is the daughter of Y and Y is the son of Z. If P is the son of Y and B is the brother of P, then

(a)  B and Y are brothers

(b)  A is the sister of B

(c)  Z is the uncle of B

(d)  Q and Y are brothers

Answer: (b)

93. Radhika went 50 m South from her house, then turned left and went 20 m, then turning to North she went 30 m. In which direction is her home from this point?

(a)  North

(b)  South-West

(c)  East

(d)  North-West

Answer: (d)

94. If South-East becomes North and North-East becomes West and all the rest directions are changed in the same manner, what will be the direction for the West?

(a)  North-East

(b)  South

(c)  South-East

(d)  South-West

Answer: (c)

Directions (Q. Nos. 95-97) In the following questions a statement is given, you have to assume everything to be true, then decide which of the suggested courses of action logically follow(s) for pursuing.

Give Answer

(a) If only course of action I follows

(b) If only course of action II follows

(c) If either course of action I or II follows

(d) If both courses of action I and II follow

95. Statement Many pilgrims died in a stampede while boarding a private ferry to the holy place on the first day of the ten-day-long festival.

Course of action

(I) The Government should immediately cancel the licences of all the private ferry operators with immediate effect.

(II) The Government should deploy an. adequate number of its personnel to guide pilgrims on their journey to the holy place.

Answer: (b)

96. Statement Some students of the local college were caught travelling in the train without purchasing valid tickets.

Courses of action

(I) The parents of these students should be informed about the incident and requested to counsel their wards.

(II) These students should be put behind bars for travelling without bona fide credentials.

Answer: (a)

97. Statement A large part of the locality was flooded as the main pipe supplying drinking water burst while the workers of a utility company were laying cables in the area.

Courses of action

(I) The civic authority should immediately arrange to repair the damage and stop loss of water.

(II) The civic authority should seek an explanation and compensation from the utility company for the damage caused by them.

Answer: (d)

Directions (Q. Nos. 98 and 99) In each of the following questions, two statements are given followed by two Conclusions I and II. You have to consider the two statements to be true even if they seem to be at variance from commonly known facts. You have to decide which of the given conclusions, if any follow from the given statements.

98. Statements

(A) Education is a process of lighting.

(B) Mind requires light to enlighten the core of cognitive aspect.

Conclusions

(I) Education is a light which removes the darkness of mind.

(II) Education is a static process for mind.

(a)  Only Conclusion I follows

(b)  Only Conclusion II follows

(c)  Both Conclusions I and II follow

(d)  Neither Conclusion I nor II follows

Answer: (a)

99. Statements

(A) Best performance in Olympics fetches a gold medal.

(B) Player ‘X’ got god medal but later was found to be using a prohibited drug.

Conclusions

(I) ‘X’ should be allowed to keep the gold medal.

(II) Gold medal should be withdrawn and given to the next person.

(a)  Only Conclusion II follows

(b)  Neither Conclusion I nor II follows

(c)  Both Conclusions I and II follow

(d)  Only Conclusion I follows

Answer: (a)

100. In this question, two statements are given followed by some conclusions. You have to decide which of the given conclusion(s) follow(s) from the given statements.

Statements

(A) There are monks among those who are felicitated for remarkable social service.

(B) Jitananda and Vidyananda are among those felicitated.

Conclusions

(I) Jitananda and Vidyananda did remarkable social service

(II) All monks do social service

(III) Jitananda and Vidyananda are not monks

(IV) All monks are felicitated

(a)  Only conclusion I follows

(b)  Only conclusion II follows

(c)  Only conclusion III follows

(d)  Only conclusion IV follows

Answer: (a)

101. The tagline ‘The Complete Man’ is associated with which of the following fashion brand?

(a)  Siyaram

(b)  Raymond

(c)  Zara

(d)  Allen Solly

Answer: (b)

102. Who is the current CEO of Indian fashion e-commerce company Myntra?

(a)  Amar Nagaram

(b)  Ananth Narayanan

(c)  Vineet Saxena

(d)  Mukesh Bansal

Answer: (a)

103. Which of the following retail stores belong to Aditya Birla Group?

(a)  Big Bazar

(b)  Croma

(c)  Hypercity

(d)  Pantaloons

Answer: (d)

104. RADO, the famous international wrist watch range belongs to

(a)  Switzerland

(b)  Spain

(c)  Japan

(d)  France

Answer: (a)

105. Which Indian film actress is the brand ambassador of online portal ‘Make My Trip’?

(a)  Priyanaka Chopra

(b)  Shraddha Kapoor

(c)  Alia Bhatt

(d)  Deepika Padukone

Answer: (c)

106. Which among the following dance forms belongs to Andhra Pradesh?

(a)  Sattriya

(b)  Bharatnatyam

(c)  Kathak

(d)  Kuchipudi

Answer: (d)

107. Which among the following is an Indian Brand?

(a)  Chanel

(b)  Zara

(c)  Gucci

(d)  Allen Solly

Answer: (d)

108. Who is the first female Chief Economist of International Monetary Fund (IMF)?

(a)  Kristalina Georgieva

(b)  Anshula Kant

(c)  Gita Gopinath

(d)  Rohini Pandey

Answer: (c)

109. Which state is known for its renowned Maheshwari and Chanderi Sarees?

(a)  Karnataka

(b)  Madhya Pradesh

(c)  Tamil Nadu

(d)  Rajasthan

Answer: (b)

110. Which of these cities is a World Heritage city by declared by UNESCO?

(a)  Ahmedabad

(b)  Delhi

(c)  Chennai

(d)  Hyderabad

Answer: (a)

111. In which state is the Manas National Park located?

(a)  Assam

(b)  Rajasthan

(c)  Meghalaya

(d)  Manipur

Answer: (a)

112. Baggit is a well known brand which sells

(a)  bags

(b)  belts

(c)  wallets

(d)  All of these

Answer: (d)

113. Who became the first ever Indian to win a gold at Badminton World Championship?

(a)  PV Sindhu

(b)  Saina Nehwal

(c)  P Kashyap

(d)  Jwala Gutta

Answer: (a)

114. Where is Louis Vuitton headquarters located?

(a)  London

(b)  Milan

(c)  Paris

(d)  New York

Answer: (c)

115. Which is the world’s biggest retailer brand?

(a)  Target

(b)  Walmart  

(c)  JC Penny

(d)  Amazon

Answer: (b)

116. Which of the following desert is located in China?

(a)  Thar desert

(b)  Atacama desert

(c)  Kalahari desert

(d)  Gobi desert

Answer: (d)

117. In which year was the Lakme Fashion Week first started?

(a)  2006

(b)  2008

(c)  2000

(d)  1999

Answer: (d)

118. Which Constitutional Amendment Act gave Constitutional Status to Panchayati Raj Institutions?

(a)  73rd

(b)  74th

(c)  93rd

(d)  94th

Answer: (a)

119. Which of the following is not a member of the Shanghai Cooperation Organization?

(a)  India

(b)  China

(c)  Japan

(d)  Russia

Answer: (c)

120. Retail venture ‘Big Bazar’ is owned by

(a)  Future Group

(b)  Bharti Group

(c)  Sriram Group

(d)  Rahja Group

Answer: (a)

121. Which of these river is a tributary of Ganga?

(a)  Son

(b)  Narmada

(c)  Godavari

(d)  Mahi

Answer: (a)

122. Which Indian American economist was awarded the Nobel Prize of Economics in 2019?

(a)  Kaushik Basu

(b)  Raghuram Rajan

(c)  Abhijeet Banerjee

(d)  Urjit Patel

Answer: (c)

123. Who among the following is not an Indian fashion designer?

(a)  Manish Malhotra

(b)  Rohit Bal

(c)  Ritu Kumar

(d)  Bibi Russell

Answer: (d)

124. Which city is known as the ‘Manchester’ of South India?

(a)  Cochin

(b)  Coimbatore

(c)  Calicut

(d)  Chennai

Answer: (b)

125. Little black dress was introduced by

(a)  Zara

(b)  Gucci

(c)  Prada

(d)  Coco Chanel

Answer: (d)

Directions (Q. Nos. 126-139) Read each case very carefully and answer the questions that follow.

CASE 1

The Chinese krait and the Chinese cobra might be the original source of the newly discovered coronavirus that has triggered an outbreak of a deadly infectious respiratory illness in China in winter 2019. The many-banded krait (Bungarus multicinctus), also known as the Taiwanese krait or the Chinese krait, is a highly venomous species of elapid snake found in much of Central and Southern China and South-East Asia.

The illness was first reported in late December 2019 in Wuhan, a major city in central China and has been rapidly spreading. Since then, sick travellers from Wuhan have infected people in China and other countries, including the United States. India, Japan, Australia, Singapore, Hong Kong.

Using samples of the virus isolated from patients, scientists in China have determined the genetic code of the virus and used microscopes to photograph it. The pathogen responsible for this pandemic is a new coronavirus. It’s in the same family of viruses as the well-known Severe Acute Respiratory Syndrome Coronavirus (SARS-CoV) and Middle East Respiratory Syndrome Coronavirus (MERS-CoV), which have killed hundreds of people in the past 17 years. The World Health Organization (WHO) has named the new coronavirus 2019-nCoV.

The name of coronavirus comes from its shape, which resembles a crown or solar corona when imaged using an electron microscope. The electron microscopic image reveals the crown shape structural details for which the coronavirus was named. This image is of the Middle East Respiratory Syndrome Coronavirus (MERS-CoV).

Coronavirus is transmitted through the air and primarily infects the upper respiratory and gastrointestinal tract of mammals and birds. Though most of the members of the coronavirus family only cause mild flu-like symptoms during infection, SARS-CoV and MERS-CoV can infect both upper and lower airways and cause severe respiratory illness and other complications in humans. This new 2019-nCoV causes similar symptoms to SARS-CoV and MERS-CoV. People infected with these coronaviruses suffer a severe inflammatory response. Unfortunately, there is no approved vaccine or antiviral treatment available for coronavirus infection. A better understanding of the life cycle of 2019-nCoV, including the source of the virus, how it is transmitted and how it replicates are needed to both prevent and treat the disease.

126. Taiwanese Krait or the Chinese Krait, the original source of the newly discovered coronavirus, is a highly venomous species of elapid snake found in much of

(a)  Central China

(b)  Southern China

(c)  South-East Asia

(d)  All of the above

Answer: (d)

127. Coronavirus affected sick travellers from Wuhan have infected people in China and other countries, including

(a)  Tanzania

(b)  Australia

(c)  Uganda

(d)  Congo

Answer: (b)

128. The Pathogen responsible for the pandemic of China is a new coronavirus. The WHO has named the new coronavirus 2019 as

(a)  SARS-CoV

(b)  MERS-CoV

(c)  nCoV

(d)  W-CoV

Answer: (c)

129. The electron microscopic image of the virus depicts which shape and structure of coronavirus?

(a)  Helical shape

(b)  Icosahedral shape

(c)  Prolate shape

(d)  Crown shape

Answer: (d)

130. Severe Acute Respiratory Syndrome Coronavirus and Middle East Respiratory Syndrome Coronavirus can infect

(a)  upper airways

(b)  lower airways

(c)  Both (a) and (b)

(d)  None of these

Answer: (c)

CASE 2

The Sveriges Riksbank Prize in Economic Sciences in Memory of Alfred Nobel 2019 (commonly known as the Nobel Prize for economics) has been awarded to Abhijeet Banerjee, Esther Duflo and Michael Kremer “for their experimental approach to alleviating global poverty.”

Through the award, the Nobel committee recognized both the significance of development economics in the world today and the innovative approaches developed by these three economists.

Esther Duflo is only the second woman to be awarded the prize (after Elinor Ostrom in 2009). Banerjee, who is also her husband, is the third ever non-white recipient (after Arthur Lewis in 1979 and Amartya Sen in 1998).

In a recent issue of the journal ‘Nature’, Goran Hansson, head of the Royal Swedish Academy of Sciences that awards the Nobel, highlighted measures to address the imbalance in gender and ethnicity among winners.

He said “we are making sure to elect women to the academy” from which the prize-awarding committees for the chemistry, physics and economics Nobels are drawn. The first woman to win the John Bates Clark Medal for top economists under 40, an important indicator of who will be awarded the economics Nobel in the future, Susan Athey, only did so in 2007.

Esther Duflo was the second winner in 2010. Since then, women winners of the Clark medal have been more frequent.

Of course, award decisions are made strictly on significance of contributions. But, based on this evidence, perhaps Athey, Amy Finkelstein (who won the medal in 2012) and Emi Nakamura (who won it in 2019) will not be far behind.

131. The Sveriges Riksbank Prize in Economic Sciences has been awarded to

(a)  Abhijeet Banerjee

(b)  Esther Duflo

(c)  Michael Kremer

(d)  All of these

Answer: (d)

132. Esther Duflo is also only the second woman to be awarded the Nobel Prize in Economics after

(a)  Goran Hansson

(b)  Elinor Ostrom

(c)  Susan Athey

(d)  Emi Nakamura

Answer: (b)

133. Abhijeet Banerjee is the third ever non-white recipient of Nobel Prize for Economics after

(a)  Arthur Lewis and Amartya Sen

(b)  Goran Hansson and Amartya Sen

(c)  Elinor Ostrom and Amartya Sen

(d)  Susan Athey and Emi Nakamura

Answer: (a)

134. The first woman to win the John Bates Clark Medal for top economists under 40 is

(a)  Susan Athey

(b)  Goarn Hasson

(c)  Elinor Ostrom

(d)  None of these

Answer: (a)

CASE 3

In the 16th century, an age of great marine and terrestrial exploration, Ferdinand Magellan led the first expedition to sail around the world. As a young Portuguese noble, he served the king of Portugal, but he became involved in the quagmire of political intrigue at court and lost the king’s favor.

After he was dismissed from service by the king of Portugal, he offered to serve the future Emperor Charles V of Spain.

A papal decree of 1493 had assigned all land in the New World, West of 50 degrees W longitude to Spain and all the land East of that line to Portugal. Magellan offered to prove that the East Indies fell under Spanish authority. On 20th September, 1519, Magllan set sail from Spain with five ships.

More than a year later, one of these ships was exploring the topography of South America in search of a water route across the continent. This ship sank, but the remaining four ships searched along the Southern peninsula of South America.

Finally they found the passage they sought near 50 degrees S latitude. Magellan named this passage the ‘Strait of All Saints’, but today it is known as the “Strait of Magellan”.

One ship deserted while in this passage and returned to Spain, so fewer sailors were privileged to gaze at that first panorama of the Pacific Ocean. Those who remained, crossed the meridian, now known as the International Date Line in the early spring of 1521 after 98 days in the Pacific Ocean. During those long days at sea, many of Magellan’s men died of starvation and disease.

Later, Magellan became involved in an insular conflict in the Philippines and was killed in a tribal battle. Only one ship and 17 sailors under the command of the Basque navigator Elcano survived to complete the westward journey to Spain and thus prove once and for all that the world is round, with no precipice at the edge.

135. Which of the following century is known as ‘an age of great marine and terrestrial exploration?

(a)  17th century

(b)  18th century

(c)  16th century

(d)  15th century

Answer: (c)

136. After being dismissed from service of Portuguese king, Magellan served under which king?

(a)  Charles V of Britain

(b)  Charles V of Spain

(c)  George IV of Britain

(d)  Czar of Russia

Answer: (b)

137. The passage which was discovered by Magellan is known by which name today?

(a)  Strait of Hormuz

(b)  Strait of Gibraltar

(c)  Strait of Magellan

(d)  Strait of Charles

Answer: (c)

138. In which year, Spanish Ships crossed International Date Line

(a)  1525

(b)  1500

(c)  1498

(d)  1521

Answer: (d)

139. Under whose command, the only surviving ship returned to Spain and proved that world is round?

(a)  Ferdinand Magellan

(b)  Elcano

(c)  Christopher Columbus

(d)  John Cabot

Answer: (b)

Directions (Q. Nos. 140-150) In each of these questions a passage is followed by several inferences. You have to examine each inferences separately in the context of the passage and decide upon its degree of truth or falsity.

(a) If the inference is ‘definitely true’ i.e. it directly follows from the facts given in the passage.

(b) If the inference is ‘probably true’ i.e. through not definitely true in the light of facts given.

(c) If you think the inference is ‘probably false’ though not definitely false in the light of facts given.

(d) If you think the inference is ‘definitely’ false’ i.e. it contradicts the given facts.

CASELET 1

Sanitation is one of the biggest problems in India. There are about 700 million people who have no access to toilets at home. Slum areas do not have toilets. People are thus forced to defecate in the open, which causes numerous diseases like diarrhoea, cholera, dehydration etc. Many rural schools also have no toilets, because of which parents do not send their kids, especially girls, to school.

Due attention was drawn towards this problem by Gandhiji but nothing much was done. A growing population is the biggest challenge causing these problems.

For example, the sewage system in Delhi was designed to meet the needs of a population of three million people. But Delhi now has more than 14 million of population. This is not just the case of Delhi; every state and region in India is the same. Though 12 million toilets claim to have been built under Swachh Bharat Abhiyan in the last five years, as per a UN report, 44% of the population continues to defecate in the open. Sanitation, solid waste management and drainage continue to pose challenges.

A ranking exercise was taken up by the Government of India called ‘Swachh Survekshan” to assess rural and urban areas for their levels of cleanliness and active implementation of Swachhata mission initiatives in a timely and innovative manner.

The objective of the survey is to encourage large scale citizen participation and create awareness amongst all sections of society about the importance of working together towards making towns and cities a better place to live in. Additionally, the survey also intends to foster a spirit of healthy competition among towns and cities to  improve their service delivery to citizens, towards creating cleaner cities and towns.

140. A growing population is the biggest challenge causing problems related to sanitation.

Answer: (a)

141. There is a lack of good hygiene habits and inadequacy of social support in society that made sanitation a major problem in the country.

Answer: (b)

142. Objective of ‘Swachh Survekshan Mission’ is to encourage people to go for cleanliness activities individually and neglect group activities

Answer: (d)

143. ‘Swachh Survekshan’ exercise to assess rural and urban areas for their levels of cleanliness has increased the healthy competition among states.

Answer: (a)

144. Most of the cities are doing good in terms of sanitation except few such as Delhi and Mumbai.

Answer: (c)

CASELET 2

Availability of clean cooking fuel is a major challenge for rural households in India. This deprivation is predominantly suffered by women and children as they are the most exposed to harmful effects of smoke from burning unclean fuels in the households.

In order to safeguard the health of women and children by providing them with a clean cooking fuel-LPG, the Government of India (GoI) launched (1st May, 2016) Pradhan Mantri Ujjwala Yojana (PMUY). The scheme aims to provide five crore deposit-free LPG connections to women belonging to Below Poverty Line (BPL) households appearing in Socio-Economic and Caste Census (SECC2011) and suffering with at least one deprivation.

In SECC-2011 Census survey, Ministry of Rural Development (MoRD) enumerated 24.49 crore (17.97 crore rural and 6.52 crore urban) households in the country. Out of these, 10.31 crore households {8.72 crore rural (48.53 per cent) and 1.59 crore urban (24.39 per cent)} suffered with at least one deprivation, which were identified by the GoI for release of five crore LPG connections under PMUY.

PMUY provides that an amount of Rs. 1600 per LPG connection towards security deposit for LPG Cylinder, Pressure Regulator and Installation Charges etc. would be borne by the Government as one-time financial assistance to adult women of BPL households included in the SECC-2011 and who are not already having any LPG connection in their household. It also prescribed that the Oil Marketing Companies (OMCs) viz. Indian Oil Corporation Limited (IOCL), Bharat Petroleum Corporation Limited (BPCL) and Hindustan Petroleum Corporation Limited (HPCL) would provide an option to PMUY beneficiaries to opt for loan, if they so desire, to cover the cost of a cooking stove and first refill. EMI of loan amount would be recovered by the OMCs from the subsidy amount due to the beneficiaries on refills.

145. It was women and children who are mostly exposed due to burning of unclean fuel in household.

Answer: (a)

146. Giving gas cylinder in the name of women will strengthen their economic status in the family.

Answer: (b)

147. Though cylinder will be provided free of cost, the charges of pressure regulator and installation charges will be borne by beneficiary.

Answer: (d)

148. Putting cost of cooking stove and refill on beneficiary will become hindrace for BPLs to go for refilling of cylinders.

Answer: (b)

149. The major challenge for rural households in India is availability of cooking fuel.

Answer: (a)

150. PMUY also prescribed OMCs, IOCL, BPCL and HPCL for beneficiaries to opt for loan if they desire to cover the cost of a cooking stove and first refill.

Answer: (a)

National Institute of Fashion & Technology (NIFT) Post Graduate 2021 Question Paper With Answer Key

NIFT (Post Graduate)

National Institute of Fashion and Technology

Solver Paper 2021

1. The length, breadth and height of a room are in the ratio of 3 : 2 : 1. If its volume be 1296 m3, find its breadth.

(a)  24 m

(b)  15 m

(c)  16 m

(d)  12 m

Answer: (d)

2. Ram and Shyam run a race of 2000 m. First, Ram gives Shyam a start of 200 m and beats him by 30 s. Next, Ram gives Shyam a start of 3 min and is beaten by 1000 m. Find the time in and is beaten by 1000 m. Find the time in minute in which Ram and Shyam can run the race separately?

(a)  8 min, 10 min,

(b)  4 min, 5 min

(c)  5 min, 9 min

(d)  6 min, 9 min

Answer: (b)

3. There is a number lock with four rings. How many attempts at the maximum would have to be made before getting the right number?

(a)  104

(b)  255

(c)  104 – 1

(d)  256

Answer: (c)

4. If the curved surface area of a cone is thrice that of another cone and slant height of the second cone is thrice that of the first, find the ratio of the area of their base.

(a)  9 : 1

(b)  81 : 1

(c)  3 : 1

(d)  27 : 1

Answer: (b)

5. Seven equal cubes each of side 5 cm are joined end to end. Find the surface area of the resulting cuboid.

(a)  750 cm2

(b)  1500 cm2

(c)  2250 cm2

(d)  700 cm2

Answer: (a)

6. 400 students took the mock test 60% of the boys and 80% of the girls cleared the cut off in the test. If the total percentage of students qualifying is 65%, how many girls appeared in the test?

(a)  100

(b)  120

(c)  150

(d)  300

Answer: (a)

7. Two fair dices are thrown. Given that, the sum of the dice is less than or equal to 4, find the probability that only one dice shows two.

(a)  1/4

(b)  1/2

(c)  2/3

(d)  1/3

Answer: (d)

8. There are 10 persons P1, P2, ….,P9, P10. Out of these 10 persons, 5 persons are to be arranged in a line such that in each arrangement P1 must occur whereas P4 and P5 do not occur. The number of such possible arrangements are

(a)  7C4 × 5!

(b)  9C5 × 5!

(c)  8C5 × 5!  

(d)  9C5 × 4!

Answer: (a)

9. In a business, A and C invested amounts in the ratio 2 : 1, whereas the ratio between amounts invested by A and B was 3 : 2. If Rs. 157300 was their profit, how much amount did B receive?

(a)  Rs. 24200

(b)  Rs. 36300

(c)  Rs. 48400

(d)  Rs. 72600

Answer: (c)

10. What will be the ratio of petrol and kerosene in the final solution formed by mixing petrol and kerosene that are present in three vessels in the ratio 4 : 1, 5 : 2 and 6 : 1, respectively?

(a)  166 : 22

(b)  83 : 22

(c)  83 : 44

(d)  None of these

Answer: (b)

11. A cistern contains 50 L of water. 5 L of water is taken out of it and replaced with wine. The process is repeated again. Find the proportion of wine and water in the resulting mixture.

(a)  1 : 4

(b)  41 : 50

(c)  19 : 81

(d)  81 : 19

Answer: (c)

12. A person saves 6% of his income, 2 yr later, his income shoots up by 15% but his savings remain the same. Find the hike in his expenditure (in approx.. per cent).

(a)  13.65%

(b)  12.45%

(c)  14.85%

(d)  15.95%

Answer: (d)

13. A reduction in the price of petrol by 10% enables a motorist to buy 5 gallons more for Rs. 180. Find the original price of petrol (in Rs. Per gallon).

(a)  20

(b)  30

(c)  40

(d)  50

Answer: (c)

14. A shopkeeper allows a discount of 10% on the marked price of an item but charges a sales tax of 8% on the discounted price. If the customer pays Rs. 680.40 as the price including the sales tax, then what is the marked price of the item?

(a)  Rs. 630

(b)  Rs. 700

(c)  Rs. 780

(d)  None of these   

Answer: (b)

15. Ashok borrows Rs. 1500 from two moneylenders. He pays interest at the rate of 12% per annum for one loan and at the rate of 14% per annum for the other. The total interest he pays for the entire year is Rs. 186. How much does he borrow at the rate of 12%?

(a)  Rs. 1200

(b)  Rs. 1300

(c)  Rs. 1400

(d)  Rs. 1000

Answer: (a)

16. A driver of an auto-rickshaw makes a profit of 20% on every trip when he carries three passengers and the price of petrol is Rs. 30/L. Find the percentage profit for the same journey if he goes for four passengers per trip and the price of petrol reduces Rs. 24/L? (Assume that revenue per passenger is the same in both the cases).

(a)  33.33%

(b)  65.66%

(c)  100%

(d)  Data inadequate

Answer: (c)

17. After receiving two successive rises, Ajitha’s salary became equal to 15/8 times of her initial salary. By how much per cent was the salary rised the first time if the second rise was twice as high (in per cent) as the first?

(a)  15

(b)  20

(c)  25

(d)  30

Answer: (d)

18. Find the principal if compound interest is charged on the principal at the rate of  per annum for two years and the sum becomes Rs. 196.

(a)  Rs. 140

(b)  Rs. 154

(c)  Rs. 150

(d)  None of these

Answer: (d)

19. A tank of 4800 m3 capacity is full of water . The discharging capacity of the pump is 10 m3/min higher than its filling capacity. As a result the pump needs 16 min less to discharge the fuel, then to fill up the tank. Find the filling capacity of the pump.

(a)  50 m3/min

(b)  25 m3/min

(c)  55 m3/min

(d)  None of these

Answer: (a)

20. Divide Rs. 6000 into two parts, so that simple interest on the first part for 2 yr at 6% per annum may be equal to the simple interest on the second part for 3 yr at 8% per annum?

(a)  Rs. 4000, Rs. 2000

(b)  Rs. 5000, Rs. 1000

(c)  Rs. 3000, Rs. 3000

(d)  None of these

Answer: (a)

21. A part of Rs. 38800 is lent out at 6% per six months. The rest of the amount is lent out at 5% per annum after 1 yr. The ratio of interest after 3 yr from the time when first amount was lent out is 5 : 4. Find the second part that was lent out at 5%.

(a)  Rs. 26600

(b)  Rs. 28800

(c)  Rs. 7500

(d)  Rs. 28000

Answer: (b)

22. A person draws a card from a pack of 52, replaces it and shuffles it. He continues doing it until he draws a heart. What is the probability that he has to make 3 trials?

(a)  9/64

(b)  3/64

(c)  5/64

(d)  1/64

Answer: (a)

23. The cost of manufacture of an article is made up of four components A, B, C and D which have a ratio of 3 : 4 : 5 : 6, respectively. If there are respective changes in the cost of +10%, −20%, −30% and +40%, then what would be the percentage change in the cost?

(a)  2.22%

(b)  1.80%

(c)  3.28%

(d)  0.95%

Answer: (a)

24. A railway passenger counts the telegraph poles on the rail road as he passes them. The telegraph poles are at a distance of 50 m. What will be his count in 4 h, if the speed of the train is 45 km/h?

(a)  600

(b)  2500

(c)  3600

(d)  5000

Answer: (c)

25. In a zoo, there are healthy deers and ducks. If the heads are counted, there are 180, while the legs are 448. What will be the number of deers in the zoo?

(a)  156

(b)  68

(c)  22

(d)  44

Answer: (d)

26. Aditi bought an article and spent Rs. 110 on its repairs. She then sold it to Samir at a profit of 20%. Samir sold it to Vikas at a lost of 10%. Vikas finally sold it for Rs. 1188 at a profit of 10%. How much did Aditi pay for the article?

(a)  Rs. 890

(b)  Rs. 1000

(c)  Rs. 780

(d)  Rs. 840

Answer: (a)

27. Mukesh borrows a certain sum of money from the ABC Bank at 10% per annum at compound interest. The entire debt is discharged in full by Mukesh on payment of two equal amounts of Rs. 1000 each, one at the end of the first year and the other at the end of the second year. What is the approximate value of the amount borrowed by him?

(a)  Rs. 1852

(b)  Rs. 1694

(c)  Rs. 1736

(d)  Rs. 1792

Answer: (c)

28. After selling a watch, Sultan found that he had made a loss of 10%. He also found that had he sold it for Rs. 27 more, he would have made a profit of 5%. The actual initial loss was what percentage of the profit earned, had he sold the watch for a 5% profit?

(a)  23%

(b)  150%

(c)  180%

(d)  200%

Answer: (d)

29. A cistern is normally filled in 6 h but takes 4 h longer to fill because of a leak in its bottom. If the cistern is full, the leak will empty it in how much time?

(a)  20 h

(b)  15 h

(c)  23 h

(d)  17 h

Answer: (b)

30. A and B completed a work together in 5 days. Had A worked at twice the speed and B at half the speed, it would have taken them 4 days to complete the job. How much time would A alone take to do the work?

(a)  10 days

(b)  20 days

(c)  15 days

(d)  25 days

Answer: (a)

Directions (Q. Nos. 31-33) In each of the following questions, an idiomatic expression/proverb has been given followed by some alternatives. Choose the one that best expresses the meaning of the given bold idiom/proverb.

31. Don’t mix with the bad hats.

(a)  people of bad character

(b)  people selling bad hats

(c)  people of poor status

(d)  people with bad hats

Answer: (a)

32. The scientist worked for donkey’s years to arrive at the formula.

(a)  a short time

(b)  for donkeys

(c)  for few years

(d)  a long time

Answer: (d)

33. The President gave away the prizes to the winners.

(a)  released

(b)  gave back

(c)  distributed

(d)  let go

Answer: (c)

Directions (Q. Nos. 34 and 35) In each of the questions below, only one among the given alternatives is correctly spelt. Find out the word with the correct spelling.

34.

(a)  Adulation

(b)  Adlation

(c)  Aduletion

(d)  Addulation

Answer: (a)

35.

(a)  Indipensable

(b)  Indipenseble

(c)  Indispansible

(d)  Indispensable

Answer: (d)

Directions (Q. Nos. 36 and 37) In each of the questions below, choose the wrongly spelt word.

36.

(a)  Potassium

(b)  Possibility

(c)  Parasology

(d)  Preamble

Answer: (c)

37.

(a)  Sanguinery

(b)  Itinerary

(c)  Temporary

(d)  Necessary

Answer: (a)

Directions (Q. Nos. 38 and 39) From amongst the options given below each word, choose the appropriate singular form.

38. Lice

(a)  Lices

(b)  Louse

(c)  Licen

(d)  Leece

Answer: (b)

39. Loaves

(a)  Loaf

(b)  Loav

(c)  Loave

(d)  Loafs

Answer: (a)

Directions (Q. Nos. 40 and 41) From amongst the options given below each word, choose the appropriate plural form.

40. Chateau

(a)  Chateaus

(b)  Chateaux

(c)  Chatea

(d)  Chateaues

Answer: (b)

41. Nucleus

(a)  Nucleuses

(b)  Nucleus

(c)  Nuclei

(d)  Nucleius

Answer: (c)

Directions (Q. Nos. 42-45) In each of these questions, choose the alternative which can replace the underlined word without changing the meaning of the sentence.

42. Tax evasion amounts to larcenous accumulation of public money by some unscrupulous individuals.

(a)  miserly

(b)  extravagant

(c)  theft

(d)  shrewd

Answer: (c)

43. I cannot believe in the veracity of his statement.

(a)  truth

(b)  usefulness

(c)  sincerity

(d)  falsity

Answer: (a)

44. People thronged to pay homage to the departed leader.

(a)  humility

(b)  tribute

(c)  obedience

(d)  allegiance

Answer: (b)

45. He is a sycophant who tries to win over politicians.

(a)  a psychologist

(b)  an opportunist

(c)  an unscrupulous man

(d)  a flatterer

Answer: (d)

Directions (Q. Nos. 46-49) Fill in the blanks.

46. Did the boys turn ……… for football practice?

(a)  up

(b)  on

(c)  back

(d)  in

Answer: (a)

47. Corruption is a standing hindrance ………. The nation’s development.

(a)  of

(b)  over

(c)  to

(d)  upon

Answer: (c)

48. You ……… mad if you think I’m going to show my answer-sheet.

(a)  are supposed to be

(b)  must be

(c)  will be

(d)  ought to be

Answer: (b)

49. At one time Mr. Nigel ……… this supermarket.

(a)  was owing

(b)  used to own

(c)  had owned

(d)  owned

Answer: (d)

Directions (Q. Nos. 50-53) In each of these questions, choose the alternative which is closest to the opposite in meaning to the underlined word.

50. There was marked deterioration in his condition.

(a)  reformation

(b)  amendment

(c)  improvement

(d)  revision

Answer: (c)

51. I find his views repugnant.

(a)  amiable

(b)  repulsive

(c)  amoral

(d)  apolitical

Answer: (a)

52. There was not a single bibulous adventurer in our expedition.

(a)  fearful

(b)  cowardly

(c)  sober

(d)  unenergetic

Answer: (c)

53. This author has perspicuity in his style.

(a)  frankness

(b)  bluntness

(c)  obtuseness

(d)  vivacity

Answer: (c)

Directions (Q. Nos. 54 and 55) In each of the following questions, choose the alternative that can be substitutes for the given word/sentence.

54. A person who eats too much

(a)  Glutton

(b)  Nibbler   

(c)  Cannibal

(d)  Omnivore

Answer: (a)

55. Fear of spiders

(a)  Hydrophobia

(b)  Social Phobia

(c)  Agoraphobia

(d)  Arachnophobia

Answer: (d)

Directions (Q. Nos. 56-75) Read the following passages and answer the questions that follow.

PASSAGE 1

Organisations are institutions in which members compete for status and power. They compete for the resources of the organization, for example, finance to expand their own departments, for career advancement and for power to control the activities of others. In pursuit of these aims, groups are formed and sectional interests emerge. As a result, policy decisions may serve the ends of the political and career systems rather than those of the concern. In this way, the goals of the organization may be displaced in favour of sectional interests and individual ambition. These preoccupations sometimes prevent the emergence of organic systems. Many of the electronics firms in their study had recently created research and development departments employing highly qualified and well-paid scientists and technicians. Their high pay and expert knowledge were sometimes seen as a threat to the established order of rank, power and privilege. Many senior managers had little knowledge of the technicalities and possibilities of new developments and electronics. Some felt that close cooperation with the experts in an organic system would reveal their ignorance and show that their experience was now redundant.

56. The author makes out a case for

(a)  organic system

(b)  research and development in organizations

(c)  an understanding between senior and middle level executives

(d)  a refresher course for senior

Answer: (a)

57. The theme of the passage is

(a)  groupism in organisations

(b)  individual ambitions in organizations.

(c)  frustration of senior managers.

(d)  emergence of sectional interests in organizations.

Answer: (d)

58. Policy decision in organization should involve

(a)  cooperation at all levels in the organization.

(b)  modernization of the organization.

(c)  attracting highly qualified personnel.

(d)  keeping in view the larger objectives of the organization.

Answer: (b)

59. “Organic system” as related to the organization implies is

(a)  growth with the help of expert knowledge.

(b)  growth with inputs from science and technology

(c)  steady all-round development.

(d)  natural and unimpeded growth.

Answer: (a)

60. The author tends to see the senior managers as

(a)  ignorant and incompetent.

(b)  a little out of step with their work environment.

(c)  jealous of their younger colleagues.

(d)  robbed of their rank, power and privilege

Answer: (a)

PASSAGE 2

Teaching more than most other professions, has been transformed during the last hundred years from a small, highly skilled profession concerned with a minority of the population, to a large and important branch of public service. The profession has a great and honourable tradition, extending from the dawn of history until recent times, but any teacher in the modern world who allows himself to be inspired by the ideals of his predecessors is likely to be made sharply aware that it is not his function to teach what he thinks, but to instill such beliefs and prejudices as are thought useful by his employers.

61. In ancient times, the teaching profession was

(a)  reserved for the upper class.

(b)  reserved for a privileged few.

(c)  open to everyone irrespective of any differences

(d)  limited to a highly skilled minority.

Answer: (d)

62. The phrase ‘Extending from the drawn of history’ indicates

(a)  tradition

(b)  culture

(c)  antiquity

(d)  All of these

Answer: (d)

63. In modern times, a successful teacher is primarily supposed to

(a)  impart knowledge.

(b)  impart new and latest skills.

(c)  toe the lines preferred by those in authority.

(d)  instill values he cherishes the most.

Answer: (c)

64. The modern teacher is not able to follow the ideals of his predecessors because

(a)  of more interest in politics than in academic activity.

(b)  of tremendous advancements in professional skills.

(c)  of social and financial constraints.

(d)  the students are not serious about studies.

Answer: (c)

65. The author seems to

(a)  be against the current trend in the teaching profession.

(b)  approve the recent developments in the mode of teaching.

(c)  be a traditionalist in his views.

(d)  consider education as a part of public service.

Answer: (a)

PASSAGE 3

Speech is a great blessing but it can also be a great curse, for while it helps us to make our intentions and desires known to our fellows, it can also, if we use it carelessly, make our attitude completely misunderstood. A slip of the tongue, the use of an unusual word or of an ambiguous word, and so on, may create an enemy where we had hoped to win a friend. Again, different classes of people use different vocabularies, and the ordinary speech of an educated man may strike an uneducated listener as pompous. Unwittingly, we may use a word which bears a different meaning to our listener from what it does to men of our own class. Thus speech is not a gift to use lightly without thought, but one which demands careful handling. Only a fool will express himself alike to all kinds and conditions of men.

66. Speech can be a curse, because it can

(a)  hurt others.

(b)  lead to carelessness.

(c)  create misunderstanding.

(d)  reveal our intentions

Answer: (c)

67. A “slip of the tongue” means something said

(a)  unintentionally.

(b)  without giving proper thought.

(c)  to hurt another person.

(d)  Both (a) and (b)

Answer: (d)

68. While talking to an uneducated person, we should use

(a)  ordinary speech.

(b)  his vocabulary.

(c)  simple words.

(d)  polite language.

Answer: (b)

69. If one used the same style of language with everyone, one would sound

(a)  flat

(b)  boring

(c)  foolish

(d)  democratic

Answer: (c)

70. The best way to win a friend is to avoid

(a)  irony in speech.

(b)  pomposity in speech.

(c)  verbosity in speech.

(d)  ambiguity in speech.

Answer: (d)

PASSAGE 4

If you are running out of ideas for your New Year’s resolution, consider running. Researchers have found that runners show greater functional connectivity in brain regions important for tasks such as planning and decision-making. “These activities, such as running, that people consider repetitive actually involve many complex cognitive functions – like planning and decision-making – that may have effects on the brain,” said one of the researchers, David Raichlen, associate professor at University of Arizona, at Tucson in the United States. For the study, the researchers compared brain scans of young adults engaged in cross-country running to young adults who do not engage in regular physical activity.

Participants were roughly of the same age – 18 to 25 – with comparable body mass index and educational levels. The runners, overall showed greater functional connectivity or connections between distinct brain regions within several areas of the brain, including the frontal cortex, which is important for cognitive functions such as planning decision-making and the ability to switch attention between tasks.

71. Research has shown that running

(a)  may weaken your heart.

(b)  could deteriorate your eye-sight.

(c)  improves your brain-functioning.

(d)  has no effect on your health.

Answer: (c)

72. Cognitive functioning of your brain impacts your

(a)  ability to take decisions.

(b)  capability to plan things.

(c)  ability to shift  your attention from one task to another.

(d)  All of the above

Answer: (d)

73. Generally, running is considered

(a)  not a good physical activity.

(b)  an activity which repeats itself regularly.

(c)  a useless activity for health.

(d)  None of the above

Answer: (b)

74. While selecting participants to conduct research, it was ensured that participants were of similar

(a)  academic background.

(b)  age group.

(c)  body mass index.

(d)  All of these

Answer: (d)

75. Which of the following statements is not true?

(a)  Running hampers various connections between different brain regions.

(b)  Running is recommended as a good hobby for people.

(c)  Participants for research included those who took part in cross-country running.

(d)  All of the above

Answer: (d)

Directions (Q. Nos. 76-79) Each of these questions has an assertion (A) and reason (R). Mark answer as

(a) if both (A) and (R) are true and (R) is the correct explanation of (A).

(b) if both (A) and (R) are true but (R) is not the correct explanation of (A).

(c) if (A) is true but (R) is false.

(d) if (A) is false but (R) is true.

76. Assertion (A) Devaluation of a currency may promote export.

Reason (R) Price of the country’s products in the international market may fall due to devaluation.

Answer: (a)

77. Assertion (A) Fiscal deficit is greater than budgetary deficit.

Reason (R) Fiscal deficit is the borrowing from the Reserve Bank of India plus other liabilities’ of the government to meet its expenditure.

Answer: (d)

78. Assertion (A) According to statistics, more female children are born each year than male children in India.

Reason (R) In India, the death rate of a male child is higher than that of the female child.

Answer: (c)

79. Assertion (A) Insect resistant transgenic cotton has been produced by inserting Bt gene.

Reason (R) The Bt gene is derived from a bacterium.

Answer: (a)

80. T is the son of P, S is the son of Q. T is married to R. R is Q’s daughter. How is S related T?

(a)  Brother

(b)  Uncle

(c)  Father-in-law

(d)  Brother-in-law

Answer: (d)

Directions (Q. Nos. 81-84) In each of these questions a statement is followed by two arguments numbered I and II.

Mark answer as

(a) if only Argument I is strong

(b) if only Argument II is strong

(c) if both the Argument are strong

(d) if neither Argument I nor II is strong

81. Statement Has the Medical Science really lengthened the span of life?

Argument I Yes, the new drugs have been able to combat the diseases and increases the span of life.

Argument II No, the eternal truth that every person’s days are numbered cannot be denied.

Answer: (a)

82. Statement Should mass media be fully controlled by the government?

Argument I Yes, the contradictory news only confuses the people.

Argument II No, its credibility will be doubtful if it confuses people.

Answer: (d)

83. Statement Should sales tax be abolished?

Argument I Yes, it will eliminate an important sources of corruption

Argument II Yes, it will bring the prices of commodities down and hence consumers will be benefited.

Answer: (b)

84. Statement Computer based technology is very fruitful for industrial development in India.

Argument I Yes, accuracy, fast production and fineness are possible through computer technology.

Argument II No, it will increase unemployment in the country.

Answer: (c)

85. A and B start walking from point O. A covers a distance of 300 m in North direction and B covers a distance of 400 m towards West. How far are they now from each other?

(a)  700 m

(b)  500 m

(c)  50 m

(d)  None of these

Answer: (b)

86. Aisha, who is Mohan’s daughter, says to Reena, “Your mother Riya is younger sister of my father, who is third child of Sanjay”. How is Sanjay related to Reena?

(a)  Uncle

(b)  Father

(c)  Grandfather

(d)  Father-in-law

Answer: (c)

87. How many 7’s not immediately preceded by 4 but immediately followed by 2 are there in the following series?

3 4 7 2 8 7 2 9 4 7 1 3 5 7 2 9 9 7 7 2 5 1 4 7 2 3

(a)  1

(b)  2

(c)  3

(d)  4

Answer: (c)

88. In a row of boys, Anand is eleventh from the left and Deepak is fifteenth from the right. When Anand and Deepak interchange their positions Anand will be fifth from the left. Which of the following will be Deepak’s position from the right after rearrangement?

(a)  Seventh

(b)  Seventeenth

(c)  Eleventh

(d)  None of these

Answer: (d)

89. Shiva walks 10 km towards North and then turns right. After walking 3 km, he again turns right and walks 7 km. Now he turns left and walks 1 km. How far is he from the starting point?

(a)  10 km

(b)  7 km

(c)  20 km

(d)  5 km

Answer: (d)

90. If in a certain language SURVIVE is coded as 9182723 and MONSOON is coded as 6549554, how is RUMOUR coded in that language?

(a)  518618

(b)  581618

(c)  865118

(d)  816518

Answer: (d)

91. Neeraj starts walking towards South. After walking 15 m, he turns towards North. After walking 20 m, he turns towards East and walks 10 m. He then turns towards South and walks 5 m. How far is he from his original position and in which direction?

(a)  10 m, East

(b)  10 m, South-East

(c)  10 m, West

(d)  10 m, North-East

Answer: (a)

92. If Neena says, “Anita’s father Raman is the only son of my father-in-law Mahipal”, then how is Bindu, who is the sister of Anita, related to Mahipal?

(a)  Niece

(b)  Daughter

(c)  Daughter-in-law

(d)  None of these

Answer: (d)

93. Thirteen students are standing in horizontal row from left to right. If all the odd numbered students in the row are shifted to successive odd numbered positions, what will be the position of Sahil who was 7th in the row initially?

(a)  9th from right

(b)  6th from left

(c)  5th from right

(d)  8th from left

Answer: (c)

94. In the following number series how many such 7’s are there which are immediately preceded by a pair of numbers whose product is more than the product of pair of numbers immediately following 7?

2 2 7 1 3 9 4 8 7 6 5 4 2 8 3 5 7 4 6 5 9 7 8 6 4 3 9 7 4 6 5 2

(a)  1

(b)  4

(c)  2

(d)  3

Answer: (d)

Directions (Q. Nos. 95-100) Complete the series replacing?

95. HSJ, IRL, JQN, ?

(a)  KPR

(b)  KPP

(c)  PKP

(d)  KOP

Answer: (b)

96. 2, 4, 7, 14, 17, 34, ?

(a)  37

(b)  39

(c)  68

(d)  70

Answer: (a)

97. 2, 4, 12, 48, 240,

(a)  960

(b)  1440

(c)  1080

(d)  1920

Answer: (b)

98. 71, 76, 69, 74, 67, 72, ?

(a)  77

(b)  80

(c)  65

(d)  76

Answer: (c)

99. In a code language, ‘mok dan sil’ means ‘nice big house’, ‘fit kon dan’ means ‘house is good’ and ‘warm tir fit’ means ‘cost is high’. Which word stands for ‘good’ in that language?

(a)  mok

(b)  dan

(c)  fit

(d)  kon

Answer: (d)

100. If in a certain language code ‘nee tim see’ means ‘how are you’, ‘ble nee see’ means ‘where are you’, then what is the code for ‘where’?

(a)  see

(b)  tim

(c)  nee

(d)  ble

Answer: (d)

101. Which company has roped in Saikhom Mirabai Chanu as its brand ambassador?

(a)  Patanjali

(b)  Amway India

(c)  Bigbasket

(d)  Modicare

Answer: (b)

102. The tagline Bleed Blue is associated with which of the following fashion brand?

(a)  Zara

(b)  Allen Solly

(c)  Nike

(d)  Adidas

Answer: (c)

103. Which of the person is the brand ambassador of Siyaram fashion brand?

(a)  Mahendra Singh Dhoni

(b)  Rohit Shetty

(c)  Virat Kohli

(d)  Ranbir Kapoor

Answer: (a)

104. Who has been roped in as the chairman of the committee formed by government to double and quadruple the exports of handlooms in 3 years?

(a)  Dinesh Bhargav

(b)  Sunil Sethi

(c)  Anamika Khanna

(d)  Ashish Soni

Answer: (b)

105. Amazon has appointed which Indian celebrity to lend voice for Alexa?

(a)  Shatrughan Sinha

(b)  Rajnikanth

(c)  Shahrukh Khan

(d)  Amitabh Bachchan

Answer: (d)

106. The ‘Jogajog’ is a platform similar to Facebook. The new platform is being developed by which country ?

(a)  Pakistan

(b)  India

(c)  Nepal

(d)  Bangladesh

Answer: (d)

107. Which of the following retail stores belong to Ambani’s Reliance Industries?

(a)  Pantaloons

(b)  Big Bazar

(c)  Shoppers Stop

(d)  Westside

Answer: (b)

108. Which of the following is an emirati retail fashion brand which comes under Dubai-based retail company?

(a)  Westside

(b)  Lifestyle

(c)  Pantaloons

(d)  Max

Answer: (b)

109. Which states is known for its renowned Tant saree and Batik saree?

(a)  Assam

(b)  Odisha

(c)  West Bengal

(d)  Gujarat

Answer: (c)

110. Which of the following brand is the famous for ladies suits in India?

(a)  Baluchari

(b)  Gucci

(c)  Flormar

(d)  Biba

Answer: (d)

111. How many Indian companies features in the 2021 Fortune’s Global 500 list?

(a)  9

(b)  5

(c)  7

(d)  11

Answer: (c)

112. Which is the first Indian city to become 100 percent COVID-19 vaccinated?

(a)  Hyderabad

(b)  Bhubaneswar

(c)  Pune

(d)  Indore

Answer: (b)

113. Neeraj Chopra has claimed the gold medal for India at Tokyo Olympics in which event?

(a)  Boxing

(b)  Shooting 

(c)  Wrestling

(d)  Javelin Throw

Answer: (d)

114. Government of India launched brand name and logo for marketing of the urban Self-Help Group (SHG) products. What brand name has been given to these products?

(a)  SonChiraiya

(b)  Neelkanth

(c)  Saras

(d)  Gauraiya

Answer: (a)

115. My e-Haat portal has been launched by which company to empower artisans?

(a)  Reliance Foundation

(b)  Infosys Foundation

(c)  Adani Foundation

(d)  HCL Foundation

Answer: (d)

116. Dholavira is the newest Indian site to be inscribed on the UNESCO’s World Heritage list. Where is Dholavira located?

(a)  Ladakh

(b)  Mizoram

(c)  Telangana

(d)  Gujarat

Answer: (d)

117. Which country has assumed the Presidency of the United Nations Security Council (UNSC)?

(a)  India

(b)  Russia

(c)  France

(d)  Germany

Answer: (a)

118. Which of the following brand belongs to Titan as sub-brand?

(a)  Chumbak

(b)  Dressberry

(c)  Fastrack

(d)  Roadster

Answer: (c)

119. Good Morning is the sub-brand of the following brands

(a)  Gucci

(b)  Park Avenue

(c)  Axe

(d)  Old Spice

Answer: (b)

120. Zodiac is the famous brand in which categories of following?

(a)  T-shirt

(b)  Jeans

(c)  Tie

(d)  Kurti

Answer: (c)

121. Who among these is the noted environmentalist and the leader of the Chipko Movement?

(a)  Mohan Dharia

(b)  Chandi Prasad Bhatt

(c)  Mike Pandey

(d)  Sunderlal Bahuguna

Answer: (d)

122. Raimona Reserve Forest has been declared as a National Park. Where is it located?

(a)  Sikkim

(b)  Assam

(c)  Tamil Nadu

(d)  Punjab

Answer: (b)

123. Who among the following has won the Miss Universe title in May 2021?

(a)  Julia Gama

(b)  Andrea Meza

(c)  Adline Castelino

(d)  Kimberly Jimenez

Answer: (b)

124. miniTV is a newly launched free video streaming service. The service has been launched by which entity?

(a)  Amazon India

(b)  Star India

(c)  Reliance

(d)  Zee Entertainment

Answer: (a)

125. The book title “Elephant In The Womb” is a debut book by which Indian celebrity?

(a)  Shilpa Shetty

(b)  Kalki Koechlin

(c)  Twinkle Khanna

(d)  Anushka Sharma

Answer: (b)

CASE 1

American scientists David Julius and Ardem Patapoutian won the 2021 Nobel Prize for Medicine for the discovery of receptors in the skin that sense temperature and touch and could pave the way for new pain-killers. Patrik Ernfors, a member of the Nobel Committee, in Oslo, Sweden, recognized the work by David Julius and Ardem Patapoutian as it has unlocked one of the secrets of nature.

Humans’ abilities to sense heat, cold, pressure and position are vital for perceiving and reacting to our surroundings. Understanding how they work is critical for treating chronic pain and other conditions. Their work, carried out independently, for discovering the molecular bases of how nerves convert stimuli-the burn of a chili pepper or the soft pressure of a hug-into signals that can be sensed by the brain.

Julius and his colleagues worked to find the receptor for capsaicin, component in chilies that causes a painful burning sensation. They identified the gene that encodes a new protein, called TRPV1-an ion channel in the membranes of cells that opens in response to heat. Julius got the idea to do his capsaicin experiments while shopping in a grocery store: “Walking through the supermarket aisle one day, seeing all these hot chili pepper sauces, et cetera, I was thinking, “We really have to get his project done,” . “And my wife said, ‘Well, then  you should get on it!”

Julius and Patapoutian independently identified another protein: TRPM8, which is sensitive to cold and menthol. Additionally, Patapoutian and his colleagues identified the genes for proteins that sense touch, known as Piezo 1 and Piezo 2. He showed that these two proteins were force activated ion channels. Piezo2 was also found to be important for sensing the positions of limbs in space, an ability known as proprioception.

126. Most of the Nobel Prizes in Medicines won by Americans and Europeans since its inception. The award is given by the Nobel Committee for Medicine based at

(a)  London

(b)  New York

(c)  Lisbon

(d)  Stockholm

Answer: (d)

127. American scientists won the Nobel Prize for the discovery of receptors in the skin that sense temperature, this will help in

(a)  New kinds of Pain Killer

(b)  Chemo Therapy

(c)  Treating Gene Structures

(d)  All of the above

Answer: (a)

128. The new identified gene that encodes a new protein, called TRPV1 found at

(a)  Berberine

(b)  Piperine

(c)  Curcumin

(d)  Capsaici

Answer: (d)

129. Julius and Patapoutian independently identified another protein named TRPM8, which is sensitive to

(a)  Cold and Menthol

(b)  Pressure and Position

(c)  Burning Sensation

(d)  All of these

Answer: (a)

130. Patapoutian and his colleagues identified the genes known as Piezol and Piezoz. Which of these two proteins were ion channels of the above two?

(a)  Shape-activated ion channels

(b)  Force-activated ion channels

(c)  Channel-activated ion channels

(d)  None of the above

Answer: (b)

CASE 2

In the year-2020, the fashion industry was trying to hold back its position, as the concept of work from home (WFH) became inevitable. Most of the people started working from home. Retail shops were shut down due to the pandemic and online shopping increased to encourage social distancing. Due to cost-cutting and new responsibilities, expensive purchases were curtailed, which subsequently affected the fashion industry.

Now that the world is slowly returning to normalcy, we have all started to regain and reshape ourselves in the new economic sphere. The designers have to come forward in terms of competitive clothing. Designer now uses Indian handloom fabric to make it more comfortable as it absorbs sweat, so it can be worn at home all day long. However, for now, designing competitive clothing is a challenge. The pandemic has taught us a huge lesson. It has given us time to rethink and reinvent ourselves.

Also, it is undeniable that clothes shape our personality, bringing out the best version of ourselves.

Also, it is undeniable that clothes shape our personality, bringing out the best version of ourselves. Our clothes and jewellery have always been inspired by our cultures and traditions. As humans started to get civilized, presenting oneself was always a prime psychological and emotional point within a human being. Moreover, the psychology of presenting oneself lies at the core of human existence.

During the 80s and 90s, Indian people were clueless about fashion. In today’s generation, our lifestyle is a reference to previous cultural influences. We need to get inspired by classical music, 500-year-old Amir Khusrow’s poetry, and Mughal influences in our craft. So, I feel that in today’s generation, our lifestyle is a reference to previous cultural influences. Even we can get inspirations from our kings and royal families, the way they were wearing their clothes, the jewellery, turbans, attire, shoes, and/or makeup.

However, we need to reinvent and make garments that are wearable and relevant in today’s time. However fashion industry in India has explored many avenues, but the real challenge is to make clothing more relevant to the modem lifestyle. Many designers has worked in each region with each design, including lehengas, cholis, anarkali, Hyderabadi and Pakistani shararas, and farshi. So, a lot of areas have already been explored.

The major question in fashion industry is how do we take out the traditional attire and make it relevant in contemporary dressing sense. Since the younger generation is fast-paced, they don’t want so much fabric and weight on themselves. One needs to select the best textile and make it into a stylish garment.

131. The Fashion Industry has evolved from normalcy to WFH culture with the concept of

(a)  Online Purchasing

(b)  Staggered Fashion

(c)  Competitive Clothing

(d)  None of these

Answer: (c)

132. Which of following have highly influenced our cultures and traditions?

(a)  Clothes and Jewellery

(b)  International Exposure

(c)  Wealth and Prosperity

(d)  Religion and Society

Answer: (a)

133. Which element/s can help us for crafting right fashion?

(a)  Classical Music and Poetry

(b)  Mughal Art and Craft

(c)  Kings and Royal Families

(d)  None of the above

Answer: (d)

134. The real challenge lies in today’s fashion industry in making relevant to

(a)  with cost cutting

(b)  modern lifestyle

(c)  contemporary and fusion

(d)  None of the above

Answer: (b)

135. What sort of fashion, younger generation wants these days?

(a)  Lighter and Stylish

(b)  Smart and Durable

(c)  Costly and Rich Work

(d)  Both (a) and (b)

Answer: (d)

CASE 3

A series of latest scams have exposed the vulnerabilities of the Aadhaar-enabled Payment System (AePS). The AePS enables a person to withdraw money from her bank account anywhere in the country using a local “business correspondent” (BC). A BC is an informal bank agent equipped with a biometric Point-of-Sale (PoS) machine-a kind of micro-ATM.

If you want to withdraw, say, Rs. 500 from your bank account using a BC, you just have to give him the name of your bank and submit yourself to Aadhaar-based biometric authentication (ABBA). The BC will give you Rs. 500 in cash, and his own account will be credited with the same amount. For this to be possible, your bank account must be linked with Aadhaar. But what if the BC enters “one thousand rupees” in the PoS machine even as he gives you five hundred? In that case, one thousand will be debited from your account, and credited to the BC’s account, but you will only get five hundred.

A corrupt BC can even get away with asking a gullible customer to put her finger in the PoS machine under some pretext, without giving her any money. Many similar cases of AePSenabled fraud have emerged in Indial. Most of them are unresolved. Even if the BC can be traced, it is easy for him to claim that he did disburse cash as per records-it is his word against the victim’s. In short, corrupt BCs operate with virtual impunity.

The AePS is a a valuable facility for those who are able to use it safely. Like other micro-A TM systems, it has helped to decongest banks. It can be particularly useful to migrant workers who have no ATM facility. But AePS can be improved through number of measures, for those who are not clear about how it works.

For instance, BCs could be required to make manual if not digital entries into printed customer passbooks. That would act as a permanent, verifiable receipt that cannot be denied to the customer so easily (a blank entry would be incriminating).

Ensuring that BCs are clearly identified in transaction records would also help. So would SMS alerts, when the customer has a mobile number. Roaming BCs should perhaps be banned, at least in states with low literacy levels. And most importantly, better grievance redressal facilities must be made available to the victims of AePS fraud.

136. A Business Correspondent (BC) is equipped with PoS machine, which help in

(a)  Withdraw money without Bank

(b)  Aadhaar-linked Payment

(c)  B2B Transfer

(d)  None of the above

Answer: (d)

137. How Aadhaar-based biometric authentication (ABBA) functions?

(a)  Through Smart Cards with chip

(b)  Through QR Code

(c)  Through biometric based Authentication

(d)  None of the above

Answer: (c)

138. The AcPS are important tool to provide assistance to poor people as it maintains

(a)  Confidentiality

(b)  Authentication

(c)  Hassle-free

(d)  None of the above

Answer: (d)

139. Various cases of AePS-enabled fraud have emerged in India because of

(a)  a corrupt BC

(b)  AePS-related irregularities

(c)  untraceable in nature

(d)  None of the above

Answer: (d)

140. The BCs can be made accountable with

(a)  Manual Entries of Details

(b)  Clear Transaction Details

(c)  Switching to Traditional Methods

(d)  Both (a) and (b)

Answer: (d)

Directions (Q. Nos. 141-150) In each of these questions a passage is followed by several inferences. You have to examine each inferences separately in the context of the passage and decide upon it degree of truth or Falsity.

(a) If the inference is ‘definitely true’ i.e. it directly follows from the facts given in the passage.

(b) If the inference is ‘probably true’ i.e. through not definitely true in the light of facts given.

(c) If you think the inference is ‘probably false’ though not definitely false in the light of facts given.

(d) If you think the inference is ‘definitely false’ i.e. it contradicts the given facts.

CASELET 1

With a combination of rapid turnover of the latest trends, affordable prices, and matchless convenience, online shopping apps are set to take over the fashion industry in India. In the last decade, e-commerce has transformed the business landscape worldwide, driven by the almost complete shift to online shopping during the Covid-19 pandemic. Sites like Ajio, owned by billionaire Mukesh Ambani’s Reliance Industries, and Mytra.com, a subsidiary of Walmart owned Flipkart, the largest player in Indian e-commerce space.

The main reasons behind the growing popularity of such platforms are competitive prices, steep discounting and true to its moniker of “fast fashion,” its ability to create and release the latest styles on demand. Though, online shopping does have its drawbacks, with numerous customers complaining about fraud pictures that show completely different colours or flawed sizing issues. These fast fashion apps may offer good deals to consumers; the speedy turnover and the short life of these garments come at big cost to the environment and ecology.

However, environmentalists are concerned about the impact of such large-scale e-commerce. The micro-plastics released from clothes are not as visible as large plastic packaging. Ingestion of these has catastrophic effects on marine life and even humans. Globally, the fashion industry is responsible for 20 pc of wastewater. To combat this, Indian designers have introduced sustainable fashion e-commerce that offers a wide range of vegan apparel created from 10 pc organic cotton in a fair-trade factory.

Fast or slow, the online fashion industry will undoubtedly continue to expand exponentially. According to IBEF, the Indian e-commerce market expected to grow from USD 46.2 billion in 2020 to USD 111.40 billion by 2025, largely led by giants such as Flipkart and Amazon. Ultimately, it may be to the consumers whether the current e-commerce giants will continue to outpace their local, smaller counterparts, or whether they will be forced to alter some of their controversially unethical manufacturing processes.

141. The growing popularity of e-Commerce platforms are due to steep discounts.

Answer: (b)

142. E-Commerce has transformed the business landscape worldwide, through completely shift to online shopping during the Covid-19 pandemic.

Answer: (a)

143. Online shopping doesn’t have any drawbacks as it offers good deals to consumers.

Answer: (d)

144. Environmentalists are concerned about e-commerce, because of different forms of environment pollution.

Answer: (a)

145. The future of E-Commerce is not bright in India as it will lead by giants like Flipkart and Amazon.

Answer: (c)

CASELET 2

Every year, millions of people are forced to abandon their homes in search of safer places to rebuild their lives. According to the UN, over 82.4 million people were force to leave their homes in 2020 and more than 20 million of them are refugees. Over 200,000 of these refuges are currently in India.

Through its history, India has hosted people fleeing war, conflict and persecution many times-Zoroastrians from Iran, Sri Lankans in the 1980s or Afghans during varied waves of displacement, including the current one in 2021. The country also has the experience of rehabilitating Partition refugees.

Welcoming refuges lies at the core of India’s secular, spiritual and cultural values. India has taken part in 49 peacekeeping missions, in which more than 195,000 troops and a significant number of police personnel assisted the UN and international NGOs in conflict-ridden lands. The paradox, therefore, of such a welcoming country not having its own homegrown national refugee framework requires a rethink.

A sustainable refugee policy is a necessary step intelligently manage population movements and ensure transparency and predictability in our administrative actions. Treatment of refugees must receive the same attention that other human rights protection issues receive-this is consistent with the constitutional emphasis on the rule of law.

The legislation will also clarify the roles of different agencies-governmental, judicial, UN – involved in refugee protection and lay down the procedures of coordination amongst them. It would also help avoid friction between the host country and the country of origin. Other states would recognize the move to grant asylum as a peaceful, humanitarian and legal act, and not an arbitrary political gesture. It will also provide a platform for dialogue on sharing responsibility and aid the search of durable solutions to the root causes of a refugee problem.

Welcoming refugees generally implies an initial investment of public funds. Once refugees start working, this investment may harvest dividends.

Refugees can fill gaps in the labour market or start trades that create wealth and help improve international trade and investment. Thanks to their diverse experiences, refugees bring new concepts, technologies and innovative ideas.

146. Accepting refuges is against the core of India’s cultural values.

Answer: (d)

147. India has given shelter to Sri Lankans in the 1980s and Afghans during the current one in 2021.

Answer: (a)

148. An urgent and temporary refugee policy is required in India to manage refugee movements.

Answer: (c)

149. In India, there is need for a legislative law to deal with the refugee crisis will and also clarify the roles of different agencies-governmental, judicial, UN involved in refugee protection.

Answer: (a)

150. If refugees shall work and invest in India, this will harvest dividends and prosperity.

Answer: (b)

National Institute of Fashion & Technology (NIFT) Post Graduate 2022 Question Paper With Answer Key

NIFT (Post Graduate)

National Institute of Fashion and Technology

Solver Paper 2022

1. If a + b – c : b + c – a : a + c – b = 5 : 6 : 7, then find a : b : c.

(a)  12 : 13 : 11

(b)  12 : 11 : 13

(c)  13 : 12 : 11

(d)  13 : 11 : 12

Answer: (b)

2. The cost price of 80 articles is Rs. 12.50 per article. 20 opf them were sold for Rs. 18 each. At what price should each of the remaining articles be sold so as to get an overall profit of Rs 4.50 per article?

(a)  Rs. 15

(b) 

(c) 

(d)  Rs. 18

Answer: (b)

3. Divide Rs 6600 into two parts so that the simple interest on the first part for 5 yr at 6% per annum is equal to the simple interest on the second part for 3 yr at 12% per annum.

(a)  Rs. 4000, Rs. 2600

(b)  Rs. 3500, Rs 3100

(c)  Rs. 3800, Rs. 2800

(d)  Rs. 3600, Rs. 3000

Answer: (d)

4. A vessel is full of a mixture of milk and water with 9% milk. 9 L are withdrawn and then replaced with pure water. If the milk is now 6%, then how much does the vessel hold?

(a)  27 L

(b)  18 L

(c)  36 L

(d)  40 L

Answer: (a)

5. There are two containers having mixtures of hydrochloric acid and water. In container 1, the ratio of hydrochloric acid and water is 1 : 2 and in container 2, the ratio of hydrochloric acid and water is 4 : 1. Find the amount of mixture that should be taken from container 1 in order to make 28 L of a mixture containing equal amount of water and hydrochloric acid.

(a)  15 L

(b)  14 L

(c)  20 L

(d)  18 L

Answer: (d)

6. P, Q and R can together earn Rs 3100 in 10 days. Q and R together can earn Rs 1320 in 6 days. P and R together can earn Rs 1050 in 5 days. Find R’s daily earning.

(a)  Rs 100

(b)  Rs 110

(c)  Rs 120

(d)  Rs 90

Answer: (c)

7. A train overtakes two persons, cycling at 9 km/h and 18 km/h in 40s and 48 s respectively. Find the length of the train.

(a)  550 m

(b)  580 m

(c)  625 m

(d)  600 m

Answer: (d)

8. A, B, C and D play four different games among Baseball, Cricket, Kabaddi and Volleyball. A does not play Baseball or Cricket. B does not play Kabaddi or Volleyball. C plays Volleyball and D plays either Baseball or Volleyball. Who plays Cricket?

(a)  A

(b)  B

(c)  C

(d)  D

Answer: (b)

9. What is the angle between the minute hand and the hour hand of a clock at 3 h 4 min ?

(a)  20°

(b)  70°

(c)  90°

(d)  130°

Answer: (d)

10. If 1st January, 1992 is a Tuesday, then on which day of the week will 1st January, 1993 fall?

(a)  Tuesday

(b)  Thursday

(c)  Friday

(d)  Saturday

Answer: (b)

11. Five villages P, Q, R, S and T situated close to each other. P is to the West of Q, R is to the South of P, T is to the North of Q and S is to the East of T. In which direction R is with respect to S?

(a)  North-West

(b)  South-East

(c)  South-West

(d)  Data inadequate

Answer: (c)

Directions (Q. Nos. 12-16) Study the following information carefully and answer the given questions.

A, B, C, D, E, F and G are seven members of a club. Each of them likes one day of the week, viz. Monday, Tuesday, Wednesday, Thursday, Friday, Saturday and Sunday, not necessarily in the same order. Each of them owns a different car, viz. Swift, Alto, Figo, Beat, SX4, Estilo and Optra, not necessarily in the same order.

C likes Wednesday and his favolurite car is neither SX4 nor Optra. E does not like Monday and his favourite car is Beat. The favourite car of one who likes Friday is Figo. The one whose favourite car of one who likes Friday is Figo. The one whose favourite car is Estilo likes Tuesday. D likes Saturday and D’s favourite car is not SX4. G’s favourite car is Alto. F likes Thursday. B does not like Estilo.

12. Who among the following likes Tuesday?

(a)  A

(b)  B

(c)  D

(d)  Data inadequate

Answer: (a)

13. Whose favourite car is Figo?

(a)  A

(b)  B

(c)  F

(d)  C

Answer: (b)

14. Who among the following likes Sunday?

(a)  A

(b)  C

(c)  F

(d)  E

Answer: (d)

15. Whose favourite car is SX4?

(a)  B

(b)  A

(c)  F

(d)  D

Answer: (c)

16. Which of the following combinations is correct?

(a)  F-Thursday-Estilo

(b)  C-Wednesday-Alto

(c)  D-Saturday-Beat

(d)  All are incorrect

Answer: (d)

Directions (Q. Nos. 17-21) Study the following information carefully and answer the given questions.

A, B, C, D, E, F and G are standing in a straight line facing North with equal distances between them, but not necessarily in the same order.

Each one is pursuing a different profession-Actor, Reporter, Doctor, Engineer, Lawyer, Teacher and Painter but not necessarily in the same order.

G is fifth to the left of C. The Reporter is third to the right of G. F is fifth to the right of A. E is second to the left of B. The Engineer is second to the left of D. There are only three people between the Engineer and the Painter. The Doctor is to the immediate left of the Engineer. The Lawyer is to the immediate right of the Teacher.

17. What is A’s profession?

(a)  Painter

(b)  Doctor

(c)  Teacher

(d)  Actor

Answer: (b)

18. Which one is true according to the given arrangement?

(a)  F is the Teacher

(b)  F is third to the left of E

(c)  The Painter is to the immediate left of B

(d)  The Lawyer is standing in the exact middle of the arrangement

Answer: (d)

19. Who among the following is an Actor?

(a)  E

(b)  F   

(c)  C

(d)  B

Answer: (c)

20. What is D’s position with respect to the Painter?

(a)  Third to the left

(b)  Second to the right

(c)  Third to the right

(d)  Second to the left

Answer: (d)

21. Who is an Engineer?

(a)  G

(b)  E

(c)  B

(d)  F

Answer: (a)

22. The first n even natural numbers is equal to k times the sum of first n odd natural numbers.

Answer: (a)

23. If 12 pumps working 7 h a day can lift 2800 tonnes of water in 20 days, then in how many days can 20 pumps working 9 h a day lift 3000 tonnes?

(a)  12

(b)  14

(c)  10

(d)  8

Answer: (c)

24. The ratio of present age of two brothers is 1 : 2 and 5 yr back the ratio was 1 : 3. What will be the ratio their age after 5 yr?

(a)  2 : 5

(b)  4 : 5

(c)  3 : 5

(d)  1 : 5

Answer: (c)

25. The price of petrol increased by 2% in a certain week and increased by 4% in the next week. Find the net percentage increase in the price of petrol over these two weeks.

(a)  6.12%

(b)  6.08%

(c)  6.16%

(d)  6.20%

Answer: (b)

26. At what per cent will simple interest on Rs 1950 amount to Rs 253.50 in 

(a)  3%

(b)  5%

(c)  4%

(d)  4.5%

Answer: (c)

27. What annual installment will discharge a debt of Rs 3094 due in 4 yr at 7% simple interest?

(a)  Rs 650

(b)  Rs 1200

(c)  Rs 900

(d)  Rs 700

Answer: (d)

28. In how many years interest of a sum will be (3/5)th of the total amount at 10% simple interest?

(a)  10  yr

(b)  12 yr

(c)  15 yr

(d)  13 yr

Answer: (c)

29. P and Q working together can complete a job in 16 days. P alone can complete it in 18 days. Both work together for 4 days and then Q leaves. Find the time taken by P to complete the remaining work.

(a)  12.5 days

(b)  13.5 days

(c)  14 days

(d)  14.5 days

Answer: (b)

30. A tank has a leak at its bottom which empties it at 6 L/min. It also has a filling tap which can fill the tank in 6 h. The tank takes 18 h to become full. Find the capacity of the tank.

(a)  3403 L

(b)  4023 L

(c)  3402 L

(d)  3240 L

[bg_colla pse view=”button-orange” color=”#4a4949″ icon=”arrow” expand_text=”Show Answer” collapse_text=”Hide Answer” ]

Answer: (d)

[/bg_collapse]

31. A person covered a certain distance at a certain speed. If his speed was 20% more, he would take 10 min less to cover the same distance. Find the time he takes to cover the distance.

(a)  60 min

(b)  50 min

(c)  45 min

(d)  42 min

Answer: (a)

32. What number will replace the question mark?

(a)  5

(b)  6

(c)  7

(d)  12

Answer: (c)

33. Which of the following countries is known as the Clove Bowl of the World?

(a)  Baharin

(b)  Zanzibar

(c)  Gibralter

(d)  Brazil

Answer: (b)

34. The land of Thousand Islands is

(a)  Malaysia

(b)  Indonesia

(c)  Ireland

(d)  Finland

Answer: (b)

35. Which of the following countries is known as Cockpit of Europe?

(a)  Sweden

(b)  Netherlands

(c)  Belgium

(d)  Germany

Answer: (c)

36. National Start Up day is

(a)  January 13

(b)  January 14

(c)  January 15

(d)  January 16

Answer: (d)

37. Where Atacama Desert is located?

(a)  Peru

(b)  Iran

(c)  Chile

(d)  Namibia

Answer: (c)

38. Who is the Managing Director of International Monetary Fund?

(a)  Kristalina Georgieva

(b)  Gita Gopinath

(c)  Geoffery W.S. Okamoto

(d)  Rodrigo Rato

Answer: (a)

39. The singer of Samaveda is known as

(a)  Ardhavayu

(b)  Udgatr

(c)  Hotra

(d)  None of these

Answer: (b)

40. The river Parushini is known as

(a)  Swati

(b)  Ravi

(c)  Ghaggar

(d)  Kabul

Answer: (b)

41. Who wrote the book “Hasanat-ul-Arifin”?

(a)  Abul Fazl

(b)  Dara Shikoh

(c)  Nizamuddin Ahmed

(d)  Abbas Khan Sherwani

Answer: (b)

42. What is the capital of Macedonia?

(a)  Vilnius

(b)  Riga

(c)  Valletta

(d)  Skopje

Answer: (d)

43. The imaginary line in southern hemisphere that can be drawn at  of equator is known as

(a)  Tropic of Cancer

(b)  Tropic of Capricorn

(c)  Antarctic Circle

(d)  Arctic Circle

Answer: (c)

44. Damon Galgut won the Booker Prize, 2021 for the book

(a)  In a Strange Room

(b)  The Good Doctor

(c)  Arctic Summer

(d)  The Promise

Answer: (d)

45. The Study of spiders and related animals is known as

(a)  Ophiology

(b)  Arachnology

(c)  Ethology

(d)  Myology

Answer: (b)

46. The capital of the country Mozambique is

(a)  Maputo

(b)  Rabat

(c)  Lilongwe

(d)  Accra

Answer: (a)

47. The Asiatic Society in Kolkata was established on

(a)  15 January 1854

(b)  25 January 1784

(c)  15 January 1874

(d)  15 January 1784

Answer: (d)

48. A car covered a certain distance at 90 km/h and returned back at 60 km/h. Find his average speed (in km/h) for the entire journey.

(a)  80

(b)  70

(c)  72

(d)  75

Answer: (c)

49. Find the time taken by a 200 m long train running at 36 km/h to cross a boy standing on a platform.

(a)  15 s

(b)  20 s

(c)  23 s

(d)  25 s

Answer: (b)

50. In a 600 m race, P gives Q a start of 200 m. Ratio of speeds of P and Q is 5 : 4. By what distance Q wins the race?

(a)  100 m

(b)  125 m

(c)  150 m

(d)  175 m

Answer: (a)

Directions (Q. Nos. 51-55) Study the following information carefully and answer the given questions.

P, Q, R, S, T, U, V and W are eight employees of a concern. Each is allotted a different locker out of eight lockers numbered 1 to 8 in a cupboard. The lockers are arranged in four rows with two lockers in each now.

Lockers 1 and 2 are in the top row from left to right, respectively while lockers 7 and 8 are in the bottom row arranged from left to right, respectively. Lockers 3 and 4 are in the second row from the top – arranged from right to left, respectively. So are lockers 5 and 6 – arranged from right to left, respectively – in the second row from the bottom. P has been allotted locker 1 while V has been allotted locker 8. T’s locker is just above that of Q which is just above that of R, whereas W’s locker is in the bottom row.

51. Which of the following cannot be the correct locker number-occupant pair?

(a)  3-Q

(b)  7-W

(c)  4-U

(d)  6-R

Answer: (d)

52. If U’s locker is not beside Q’s locker, whose locker is just above the of W’s locker?

(a)  U

(b)  S

(c)  R

(d)  Q

Answer: (a)

53. Which of these pairs cannot have lockers that are diagonally placed?

(a)  P-Q

(b)  S-R

(c)  U-R

(d)  Either option (b) or (c)

Answer: (d)

54. Which of the following groups consists only occupants of odd numbered lockers?

(a)  Q, R, W

(b)  R, V, W

(c)  T, R, Q

(d)  P, T, Q

Answer: (a)

55. If U’s locker is in the same row as that of R and S exchanges his locker with V, then who is the new neighbor of V in the same row? (assume that nothing else is distributed from the original arrangement)

(a)  P

(b)  Q

(c)  R

(d)  U

Answer: (b)

Directions (Q. Nos. 56-60) Study the following information carefully and answer the given questions.

P, Q, R, S and T sit around a table. P sits two seats to the left of R and Q sits two seats to the right of R.

56. If S sits in between Q and R, who sits to the immediate right of P?

(a)  T

(b)  S

(c)  Q

(d)  R

Answer: (a)

57. Which of the following cannot be the correct seating arrangement of the five persons in either the clockwise direction or the anti-clockwise direction?

(a)  P, Q, R, S, T

(b)  P, S, R, T, Q

(c)  P, Q, S, R, T

(d)  P, T, R, S, Q

Answer: (a)

58. If S is not sitting next to Q, who is sitting between Q and S ?

(a)  R

(b)  P

(c)  T

(d)  Both R and P

Answer: (b)

59. If a new person U joins the group such that the initial conditions for the seating arrangement should be observed and also a new condition that U does not sit next to R be satisfied, then which of the following statements is true?

(a)  U sits to the immediate right of S

(b)  U sits to the immediate left of T

(c)  U sits to the immediate left of P

(d)  Either a or b

Answer: (c)

60. If a new person U joins the group such that the initial conditions for the seating arrangement should be observed and also a new condition that U does not sit next to P, S or T be satisfied, then who will be the neighbours of P (one on either side) ?

(a)  S and T

(b)  S and Q

(c)  T and R

(d)  R and Q

Answer: (a)

Directions (Q. Nos. 61-65) Choose the one which can be substituted for the given words/sentence.

61. Commencement of adjacent words with the same letter.

(a)  Consonance

(b)  Concubine

(c)  Alliteration

(d)  Euphemism

Answer: (c)

62. A wall built to prevent the sea or a river from flooding an area-

(a)  Dyke

(b)  Surveillance

(c)  Nuance

(d)  Reconnaissance

Answer: (a)

63. An entertainer who performs difficult physical actions

(a)  Clown

(b)  Gymnast

(c)  Nuance

(d)  Acrobat

Answer: (d)

64. A white skinned woman with dark brown hair-

(a)  Amazon

(b)  Brunette

(c)  Swashbuckler

(d)  Anodyne

Answer: (b)

65. Bitter and violent attack in words-

(a)  Pandemonium

(b)  Prototype

(c)  Diatribe

(d)  Profanity

Answer: (c)

Directions (Q. Nos. 66-70) Groups of four words are given. In each group one word is wrongly spelt, Find out the wrongly spelt word.

66.

(a)  Connoisseur

(b)  Insolvent

(c)  Jubillee

(d)  Spendthrift

Answer: (c)

67.

(a)  Renunciation

(b)  Quotiant

(c)  Survivor

(d)  Efficient

Answer: (b)

68.

(a)  Commemoration

(b)  Accommodation

(c)  Asassination

(d)  Association

Answer: (c)

69.

(a)  Abarrant

(b)  Abecedarian

(c)  Barouche

(d)  Barbarian

Answer: (a)

70.

(a)  Replaceable

(b)  Presumptuous

(c)  Prerogative

(d)  Predater

Answer: (d)

Directions (Q. Nos. 71-75) Choose the one which best expresses the meaning of the given word.

71. INADVERTENT

(a)  Thoughtless

(b)  Unintentional

(c)  Insane

(d)  Unintelligent

Answer: (b)

72. INFURIATE

(a)  Burn

(b)  Disgrace

(c)  Threaten

(d)  Enrage

Answer: (d)

73. PROSPECTIVE

(a)  Potential

(b)  Prosperous

(c)  Perplexed

(d)  Possible

Answer: (a, d)

74. LOUSY

(a)  Unbearable

(b)  Unpleasant

(c)  Awful

(d)  Stinking

Answer: (c)

75. ACCENTUATED

(a)  Exhibited

(b)  Devalued

(c)  Mitigated

(d)  Sharpened        

Answer: (a)

Directions (Q. Nos. 76-80) Choose the word opposite in meaning to the given word.

76. LACERATION

(a)  Convalescence

(b)  Recuperation

(c)  Healing

(d)  Palpitation

Answer: (c)

77. IMPLICATE

(a)  Disentangle

(b)  Impersonate

(c)  Complicate

(d)  Impose

Answer: (a)

78. INGEST

(a)  Disrupt

(b)  Disgorge

(c)  Dismiss

(d)  Display

Answer: (b)

79. HERETICAL

(a)  Contradictory

(b)  Doubtful

(c)  Impious

(d)  Orthodox

Answer: (d)

80. PHILISTINE

(a)  Cultured

(b)  Libertine

(c)  Sober

(d)  Educated

Answer: (a)

Directions (Q. Nos. 81-85) In the following question a part of the sentence is bold. Below are given alternatives to the bold part as (a), (b), (c), and (d) which may improve the sentence. Choose the correct alternative.

81. The young boy was considered uncorrectable by his parents.

(a)  impolite

(b)  incorrigible

(c)  unruly

(d)  no improvement

Answer: (b)

82. Write down the address lest you may forget.

(a)  you may not forget

(b)  you cannot forget

(c)  you will forget

(d)  you should forget

Answer: (d)

83. He is addicted to smoke.

(a)  addicted to smoking

(b)  used to smoke

(c)  addicted of smoking

(d)  addicted with smoking

Answer: (a)

84. I will be with you in one quarter of an hour

(a)  a quarter of one hour

(b)  a quarter of an hour

(c)  a quarter of hour

(d)  no improvement

Answer: (b)

85. I am thinking to do an M.A. in English

(a)  wondering to do

(b)  thinking doing

(c)  thinking of doing

(d)  no improvement

Answer: (c)

Directions (Q. Nos. 86-90) Choose the alternative which best expresses the meaning of the idiom/phrase.

86. There is no love lost between any two neighbouring countries in the world.

(a)  stop loving

(b)  not on good terms

(c)  forming a group

(d)  have good understanding

Answer: (b)

87. When it was the peak season of his business, he played ducks and drakes with money.

(a)  distributed among the poor

(b)  spent money carelessly

(c)  played childish game

(d)  closed his season

Answer: (b)

88. The story of his survival in Tsunami beggars description.

(a)  is beyond description

(b)  is interesting

(c)  is described by a beggar

(d)  not described by anyone

Answer: (a)

89. I don’t know why she has become stand-offish recently.

(a)  angry

(b)  hilarious

(c)  indifferent

(d)  unmanageable

Answer: (c)

90. The president of this club should be someone who is open and above board.

(a)  far better

(b)  highly qualified

(c)  honest

(d)  rich and hard working

Answer: (c)

Directions (Q. Nos. 91-95) Read the following passage carefully and Answer the Questions given below:

Many people are aware of the famous “witch trials” that occurred at the end of the seventeenth century in Salem, Massachusetts. But fewer people know that men, women, and children had been persecuted in New England for practicing witchcraft decades before the Salem trials ever took place. The events in Salem happened between February 1692 and May 1693 and resulted in twenty deaths, but colonies in Connecticut, New Hampshire, and Vermont accused women of practicing witchcraft as early a the 1640s. Colonies in Connecticut executed eleven women on these charges between 1647 and 1663.

Colonists brought with them to North America the widespread European belief in dark magic and the supernatural. Suspicious behavior, sudden illnesses or deaths in families, and rivalries among neighbours often led to accusations of witchcraft. Witchcraft was a capital crime in colonial North America until 1750, and though the judicial procedure varied from colony to colony, often the only evidence needed to start a trial was the testimony of a single witness. The accused woman was ordered to appear before a judge to confess her wrongdoing. If she did not confess, she was brought before a jury, where she endured questioning, torture, and tests that would supposedly reveal her affiliation with the devil. If the woman was found guilty, she was sentenced to death.

The trials ceased in the mid-1690s after thirty-five people had been killed. The governor of Massachusetts put an end to the process after receiving pleas from legal scholars-though it should be noted that the governor’s own wife had been accused of witchcraft right before he called off the trials. Colonists began demanding public apologies from the judges and jury members in 1695, and petitions were filed seeking to reverse the alleged witches convictions as early as 1700. The Massachusetts, New Hampshire, and Vermont governments have formally acknowledged the harm done to the women, but the Connecticut government has yet to clear their names. To this day, descendants of the eleven women put to death in Connecticut hundreds of years ago are fighting to clear their ancestor’s names.

91. According to the passage, in which year did the Salem witch trials end?

(a)  1647

(b)  1663

(c)  1692

(d)  1693

Answer: (d)

92. Which of the following would be the most appropriate subtitle for this passage?

(a)  Seeking Justice for Connecticut’s Convicted Witches

(b)  The Lost Souls of Salem

(c)  The Witch Trials of Colonial New England

(d)  Religion and Witchcraft in Early New England

Answer: (c)

93. Which of the following words best describes the public apology given to victims of the colonial witch trials?

(a)  partial

(b)  delayed

(c)  inclusive

(d)  appropriate

Answer: (a)

94. Based on the information in the final paragraph, it can be inferred that the author believes

(a)  the trials should have continued well into the 1700s

(b)  the Massachusetts governor should have formed an improved legal system       

(c)  the descendants of the Connecticut witches are fighting a useless battle

(d)  the governor’s decision to end the trials may have been unduly influenced

Answer: (d)

95. In paragraph 2, we learn that “Colonists brought with them to North America the widespread European belief in dark magic and the supernatural.” Based on its use in paragraph 2, which of the following examples accurately describes a scenario involving the supernatural?

(a)  Ben can read books faster than anyone else in class.

(b)  Jamie told me she talks to her guardian angel every night.

(c)  The mansion on the hill has been abandoned for over a century.

(d)  Calvin expected the house to be full of people, but it was dark and silent

Answer: (b)

Directions (Q. Nos. 96-100) Read the following passage carefully and answer the questions given below.

In the height of the Enlightenment, men influenced by the new political theories of the era launched two of the largest revolutions in history. These two conflicts, on two separate continents, were both initially successful in forming new forms of government. And yet, the two conflicts, though merely a decade apart, had-radically different conclusions. How do two wars inspired by more or less the same ideals end up so completely different? Why was the American Revolution largely a success and the French Revolution largely a failure?

Historians have pointed to myriad reasons-far too various to be listed here. However, the most frequently cited are worth mentioning. For one, the American Revolution was far removed from the Old World; that is, since it was on a different continent, other European nations did not attempt to interfere with it. However, in the French Revolution, there were immediate cries for war from neighbouring nations. Early on, for instance, the ousted king attempted to flee to neighbouring Austria and the army waiting there. The newly formed French Republic also warred with Belgium, and a conflict with Britain loomed. Thus, the French had the burden not only of winning a revolution but also defending it from outside. The Americans simply had to win a revolution.

Secondly, the American Revolution seemed to have a better chance for success from the get-go, due to the fact that Americans already saw themselves as something other than British subjects. Thus, there was already a uniquely American character, so, there was not as loud a cry to preserve the British way of life. In France, several thousands of people still supported the king, largely because the king was seen as an essential part of French life. And when the king was first ousted and then killed, some believed that character itself was corrupted. Remember, the Americans did not oust a king or kill him-they merely separated from him.

Finally, there is a general agreement that the French were not as unified as the Americans, who, for the most part, put aside their political differences until after they had already formed a new nation. The French, despite their Tennis Court, Oath, could not do so. Infighting led to inner turmoil, civil war, and eventually the Reign of Terror, in which political dissidents were executed in large numbers. Additionally, the French people themselves were not unified. The nation had so much stratification that it was impossible to unite all of them-the workers, the peasants, the middleclass, the nobles, the clergy-into one cause. And the attempts to do so under a new religion, the Divine Cult of Reason, certainly did not help. The Americans, remember, never attempted to change the society at large; rather, they merely attempted to change the government.

96. It can be inferred from paragraph 1 that the author most likely believes that

(a)  historians have not come to a conclusion about what separates the French and American Revolutions

(b)  the French Revolution could have been just as successful as the American Revolution was

(c)  two events based on the same philosophy are generally not that common

(d)  two events that are similar in one way ought to be similar in other ways

Answer: (d)

97. What function do the questions that end paragraph I play in the passage?

(a)  They introduce the topic the author of the passage will explore, even though he or she will not answer the questions.

(b)  They introduce the author’s opinions on the main topic and summarise his or her main argument.

(c)  They present rhetorical questions that are used only for the purposes of engaging the reader, even though the questions will not be answered.

(d)  They refocus the passage into its original topic after a detour into historical context.

Answer: (b)

98. As used in paragraph 2, the word myriad most nearly means

(a)  an unknowable secret

(b)  an uncountable number

(c)  an unrealistic ideal

(d)  an unusually rare occurrence

Answer: (b)

99. According to the passage, the Reign of Terror happened because of

(a)  social stratification in America

(b)  infighting in the American Revolution

(c)  the decision to kill the king in the French Revolution

(d)  infighting in the French Revolution

Answer: (d)

100. According to the author, which of the following best summarises the most frequently cited explanations historians give for why the French and American Revolution did not turn out the same way?

(a)  Locations relative to Europe, social equality in America, and the lack of a unique French identity

(b)  Social unity among the Americans, social stratification among the French, and the character of the French and Americans in general

(c)  Locations relative to Europe, American and French characters, and political cohesion or lack thereof

(d)  Locations relative to Europe, the decision to oust or not oust a king, and a lack of unity amongst all the revolutionaries

Answer: (c)

Directions (Q. Nos. 101-105) Read the following passage and answer the questions given below:

By the mid-nineteenth century, mass production of paper patterns, the emergence of the home sewing machine, and the convenience of mail order catalogs brought fashionable clothing into the American home. By the early twentieth century, home economists working in extension and outreach programs taught women how to use paper patterns to improve the fit and efficiency of new garments as well as how to update existing ones.

Teachers of home economics traditionally made home sewing a critical part of their curriculum, emphasizing self-sufficiency and resourcefulness for your women. However, with the increasing availability of mass-produced clothing in catalogs and department stores, more and more women preferred buying garments to making them. As a result, home economist shifted their attention to consumer education. Through field study, analysis, and research, they became experts on the purchase and preservation of ready-to-wear clothing for the family, offering budgeting instruction targeted at adolescent girls. Modern home sewing made it possible for American women to transcend their economic differences and geographic locations with clothing that was increasingly standardized. The democratisation of fashion continued through the twentieth century as the ready-to-wear market expanded and home sewing became more of a pastime than a necessity.

101. According to the passage, the advent of mail order catalogs altered the role of home economists because

(a)  mass-produced clothing rendered their jobs obsolete

(b)  women ceased sewing so home economists had to teach other subjects

(c)  their focus shifted to instruction on budgeting and buying and preserving clothing

(d)  home economists had to compete with the ready-to-wear marketplace.

Answer: (b)

102. The passage focuses on the

(a)  historical shifts in home sewing from the mid-nineteenth century through the twentieth century

(b)  changing role of home economists as a result of changes in the world

(c)  modernization of home sewing

(d)  effects of home economists on home sewing

Answer: (b)

103. As used in paragraph 2 which is the best definition for democratization?

(a)  Transitioning to a more democratic political regime

(b)  Altering or modifying in a beneficial way

(c)  Becoming more affordable to the lower class

(d)  Becoming widely available to a populace

Answer: (d)

104. Based on information in the passage, it can be inferred that home sewing allowed American women to do all of the following except

(a)  continue to wear clothing that had gone out of style in stores

(b)  copy fashions they had seen elsewhere

(c)  create less expensive versions of current styles

(d)  become experts on budgeting and consumerism

Answer: (d)

105. According to the passage, which of the following led to a decline in home sewing?

(I) mail order catalogs

(II) the availability of apparel in retail outlets

(III) curriculum changes in home economics

(a)  Only I

(b)  Only II

(c)  Both I and II

(d)  Both II and III

Answer: (c)

Directions (Q. Nos. 106-110) Read the following passage and answer the questions given below:

Ancient Greek doctors relied heavily on the theory of humorism, a belief that the body was essentially balanced by four substances known as humours and that any ailme was simply the result of an imbalance of these humours. The four substances-black bile, yellow bile, phlegm, and blood-were all found in the body, and each was associated with a different element-air, fire, earth, and water, respectively; too much of one would cause disease. An excess of black bile in your body, for instance, meant you were melancholic; too much blood made you amorous and sanguine. While it is easy today to sneer at such beliefs, theories like this one; though ludicrous, matter historically. The beauty of the ancient wisdom is not that it was actually wise but that the logic behind it opened the door for future scientific inquiry. Thus, while my cough might not be caused by a lack of blood, I can thank Hippoerates and his cohorts for encouraging later doctors to find the actual source of my cough.

106. According to the passage, which of the following correctly matches a humour to its element?

(a)  Phlegm : fire

(b)  Black bile : water

(c)  Yellow bile : fire

(d)  Blood : earth

Answer: (c)

107. As used near the end of the passage, the phrase ‘sneer at’ most nearly means

(a)  deplore

(b)  deride

(c)  debunk

(d)  decline

Answer: (b)

108. It can be inferred from the passage that Hippocrates was

(a)  an ancient philosopher         

(b)  a disease researcher

(c)  a modern humanist

(d)  a Greek physician

Answer: (d)

109. Which of the following would be the most appropriate subtitle for this passage?

(a)  Ancient Humor, Modern Applications    

(b)  Greek Philosophy and the Origin of Science

(c)  The Humorists and the Scientists

(d)  The Humors and Why They Matter

Answer: (d)

110. It can be inferred from the penultimate (second-to-last) sentence of the passage that the author believes the hunours

(a)  deserve to be studied today for historical reasons

(b)  are laughably inaccurate as explanations for ailments

(c)  can offer alternative treatment for diseases

(d)  should continue to be taught in medical schools

Answer: (a)

Directions (Q. Nos. 111-115) Read the following passage and answer the questions given below:

Imagine a mythical beast rising before you on two legs that end in cloven hooves. Sharp, curled horns and pointed ears sit atop its head, while a long tongue lolls out of a mouth that seems perpetually frozen in a sinister smile. It carries rusty chains and a pack upon its back, and it waits until nightfall to enter homes and terrorize youngsters, For centuries, horrifying tales of this creature induced nightmares in children during a certain holiday. But while this monster seems like it would be right at home in a Halloween horror story, children across Europe know the creature as Krampus, a demonic creature that punishes misbehaving youth during Christmas.

The legend of Krampus has its roots in Germanic folklore and was popularized in central European countries such as Austria, Hungary, and Slovenia during the seventeenth century. The beast was said to be the uncharitable counterpart to the magnanimous St. Nicholas. While St. Nicholas filled children’s wooden shoes and stockings with treats, Krampus sought out the children who didn’t deserve gifts and delivered spankings and whippings. Children who had been especially bad during the year were in danger of being kidnapped and taken back to the creature’s lair for further punishment until they showed true remorse for their misbehaviour. By the mid-twentieth century, however, people were actively trying to bring an end to perpetuating the story of Krampus. Prominent newspapers across Europe published editorials discouraging parents to share the legend with their children, and the Austrian government distributed “Krampus Is an Evil Man,” a cautionary pamphlet written by psychologists. Thankfully, fewer children grow up fearing Krampus is modern times. But fans of history, folk tales, and offbeat holiday traditions still take time every December to reflect on one of the most frightening characters ever associated with a holiday.

111. Using the passage as a guide, it can be understood that

(I) efforts taken in the mid-twentieth century to discourage sharing the tale of Krampus were effective

(II) the legend of Krampus was popularized when the Austrian government distributed pamphlets

(III) children in Central European countries were afraid of getting eaten by Krampus every Christmas

(a)  I only

(b)  II only

(c)  Both I and II

(d)  Both II and III

Answer: (a)

112. Based on its use in paragraph 2, it can be inferred that the word ‘magnanimous’ belongs to which of the following word groups?

(a)  dastardly, vile, detestable

(b)  generous, compassionate, benevolent

(c)  refined, polish, genteel

(d)  astute, perceptive, intelligent

Answer: (b)

113. Which of the following statements would the author most likely agree with?

(a)  It is for the best that fewer children grow up knowing the legend of Krampus.

(b)  The image of Krampus was not very frightening and shouldn’t have bothered children.

(c)  Children should not expect gifts during the holidays unless they behave all year.

(d)  The legend of Krampus is too juvenile to be discussed in prominent newspapers.

Answer: (a)

114. In the final sentence of the passage, we learn that, “fans of history, folk tales, and offbeat holiday traditions still take time every December to reflect on one of the most frightening characters ever associated with a holiday.” Based on its use in this sentence, which of the following accurately describes something that is offbeat?

(a)  The new restaurant downtown offer simple, traditional American cuisine.

(b)  The toddler had a tantrum after his mother refused to buy him a new toy.

(c)  I ate turkey, stuffing, and pumpkin pie on Thanksgiving last year.

(d)  My cousin’s handmade clothing combines contrasting colors and patterns.

Answer: (d)

115. Which of the following would be the most appropriate subtitle for this passage?

(a)  Holiday Monsters from Halloween to Christmas         

(b)  The Yuletide Legends of Central Europe

(c)  Austria’s Holiday Traditions and Rituals

(d)  On the Dark Side of Christmas

Answer: (d)

Directions (Q. Nos. 116-120) Read the following passage and answer the questions given below:

By far, the most popular exhibit at the National Zoo in Washington, DC is the zoo’s pair of giant pandas, Mei Xiang and Tian Tian. These are the second set of pandas government made to the Nixons during then-President Nixon’s historic trip to China in 1972. Nixon was the first sitting President to visit China since the nation became Communist. The popularity of the exhibit today is easy to understand: The zoo’s giant pandas are two of only 300 captive pandas in the world, two of only 1,900 total pandas in either China or captivity! Yet, while literally thousands of people ‘bear’ witness to the giant pandas each day, few can ‘bear’ to face a simple reality: the socalled ‘panda bears’ may not be bears to all.

The debate began in 1869, when the first European witness of a giant panda likened the strange animal to a bear. However, Alphonse Milne-Edwards, a French scientist, challenged that classification after reviewing the remains of a giant panda and concluding that it was physiologically closer to the red panda, a member of the raccoon family. To this day, scientists are not certain about how to classify the giant panda.

But the giant panda is plainly a bear on sight, so how can it be classified with raccoons? Well, part of it is because taxonomy is hardly a perfect science, and classification occurs on many levels; appearance, genetics, behaviour, evolution, etc. In appearance, the red and giant pandas are as similar as they are dissimilar. Likewise, DNA research has revealed similar links between giant pandas and both bears and raccoons. Behaviourally, the only animal that is clearly similar to the giant panda is the red panda. So, even though the giant panda moves like a bear, it eats bamboo in the same manner as the red panda. Eating bamboo could, however, be a trait that developed independently in both species, meaning that tracing evolution many likewise be an inconclusive path. Perhaps, in the end it is best to merely classify both red and giant pandas as, simply, ‘pandas’. Certainly, the classification does not make the animals any less majestic.

116. Which sentence, if removed from the first paragraph, would strengthen the passage?

(a)  “By far, the most popular exhibit at the National Zoo in Washington, DC is the zoo’s pair of giant pandas. Mei Xiang and Tian Tian.”

(b)  “These are the second set of pandas the zoo has owned; the first two were a gift the Chinese government made to the Nixons during then-President Nixon’s a historic trip to China  in 1972.”

(c)  “Nixon was the first sitting president to visit China since the nation became Communist.”

(d)  “The popularity of the exhibit today is easy to understand: The zoo’s giant pandas are two of only 300 captive pandas in the world, and two of only 1,900 total pandas in either China or captivity!”

Answer: (c)

117. The author traces the popularity of the giant panda exhibit at the National Zoo to the

(a)  debate over classifying giant pandas

(b)  fact that the pandas were gifts to the United States

(c)  small number of giant pandas in the wild

(d)  scarcity of giant pandas in zoos

Answer: (d)

118. Each of the following can be inferred from the passage except that

(a)  giant pandas primarily live in China

(b)  the first giant panda was seen by a non-European

(c)  giant pandas are clearly members of the raccoon family

(d)  scientists are still not certain how to classify giant pandas

Answer: (c)

119. Which of the following statements made in the passage least summarizes the author’s main point?

(a)  “to this day, scientists are not certain about how to classify the giant panda”

(b)  “taxonomy is hardly a perfect science”

(c)  “the so-called’ panda bears’ may not be bears at all”

(d)  “it is best to merely classify both the red and giant pandas as, simply, pandas”

Answer: (b)

120. The author’s purpose in writing this passage is most likely to

(a)  suggest that a species has been misclassified

(b)  inform readers about an important scientific debate

(c)  argue that a species is difficult to classify

(d)  refute an argument about two species

Answer: (c)

© Copyright Entrance India - Engineering and Medical Entrance Exams in India | Website Maintained by Firewall Firm - IT Monteur